Odia govt jobs   »   Weekly Practice Quiz For For OSSSC...

Weekly Practice Quiz For For OSSSC RI,ARI, Amin: June-8

The Weekly Practice Quiz for OSSSC RI, ARI, and Amin is an essential tool for aspirants preparing for the Odisha Sub-ordinate Staff Selection Commission exams. These quizzes help candidates regularly assess their knowledge and understanding of the syllabus, covering topics like general awareness, mathematics, reasoning, and English. By consistently participating in these practice sessions, candidates can identify their strengths and weaknesses, improve their time management skills, and become familiar with the exam pattern. Regular practice through these quizzes enhances confidence and competence, paving the way for better performance in the actual examination.

Weekly Practice Quiz For OSSSC RI,ARI, Amin: June-8 PDF Download

Weekly Practice Quiz For For OSSSC RI,ARI, Amin: June-8

  1. ‘ଚଷୁ କୁଟିବା’ – ରୂଢିଟିର ଅର୍ଥ କ’ଣ?
    A) ବ୍ୟର୍ଥ ପରିଶ୍ରମ କରିବା
    B) ପରଖ୍
    C) ଧାନ କୁଟିବା
    D) ଉପାର୍ଜନ କରିବା
    Ans: A) ବ୍ୟର୍ଥ ପରିଶ୍ରମ କରିବା
  2. ‘ଗାଙ୍ଗେୟ’ ଶବ୍ଦର ବ୍ୟୁପ୍‌ ଗତ ଅର୍ଥ ନିମ୍ନଲିଖିତ ମଧ୍ୟରୁ କେଉଁଟି ଠିକ ଦର୍ଶାଅ ।
    A) ଗଙ୍ଗାଙ୍କର ସନ୍ତାନ
    B) ଗଙ୍ଗାମାତା
    C) ଗଙ୍ଗାପତି
    D) ଗଙ୍ଗାର ଉତ୍ପତ୍ତି
    Ans: A) ଗଙ୍ଗାଙ୍କର ସନ୍ତାନ
  3. ଭିନ୍ନଧର୍ମୀ ଶବ୍ଦଟି ଚିହ୍ନଟ କର |
    A) ରନ୍ଧନ
    B) ଜଣାଣ
    C) ଗାୟକ
    D) ଭୋଜନ
    Ans: C) ଗାୟକ
  4. ନିମ୍ନଲିଖିତ ମଧ୍ୟରୁ କେଉଁଟି ‘ଅପରାଧୀ’ ର ବିପରୀତ ଶବ୍ଦ?
    A) ନିରପରାଧ
    B) ଅର୍ଥୀ
    C) ଅପାୟ
    D) ଅପରାଜିତ
    Ans: A) ନିରପରାଧ
  5. ନିମ୍ନୋକ୍ତ ମଧ୍ୟରୁ କେଉଁଟି ଉପସର୍ଗରେ ଗଠିତ ଶବ୍ଦ ନୁହେଁ?
    A) ବାଦ
    B) ପ୍ରତିବାଦ
    C) ସଂବାଦ
    D) ଅନୁବାଦ
    Ans: A) ବାଦ
  6. ରାମବାବୁଙ୍କ ପାଖରେ ଟଙ୍କା ପଇସା କିଛି ନଥିବାରୁ ଖାଲି ହାତରେ ଘରକୁ ଫେରିଲେ ରେଖାଙ୍କିତ ଶବ୍ଦଟି କି ପ୍ରକାର ଯୁଗ୍ମ ଶବ୍ଦ?
    A) ବିପରୀତାର୍ଥକ
    B) ଦ୍ଵିରୁକ୍ତି
    C) ସମାନର୍ଥକ
    D) ସାର୍ଥକ ଓ ନିରର୍ଥକ
    Ans: C) ସମାନର୍ଥକ
  7. ନିମ୍ନଲିଖିତତ ମଧ୍ୟରୁ କେଉଁଟି ଏକ ତତ୍ ସମ ଶବ୍ଦ ଅଟେ?
    A) ପିଠି
    B) ଗଭୀର
    C) ଆଠ
    D) ଭଉଣୀ
    Ans: B) ଗଭୀର
  8. ‘ସମ୍ବନ୍ଧ ପଦ’ ରେ କେଉଁ ବିଭକ୍ତି ହୁଏ?
    A) ତୃତୀୟା
    B) ଷଷ୍ଠୀ
    C) ଚତୁର୍ଥୀ
    D) ପଞ୍ଚମୀ
    Ans: B) ଷଷ୍ଠୀ
  9. ‘ପାନିଆଁ’- କେଉଁ ଶ୍ରେଣୀ ର ଶବ୍ଦ ଅଟେ?
    A) ବୈଦେଶିକ
    B) ତଦ୍ ଭବ
    C) ତତ୍ ସମ
    D) ଦେଶଜ
    Ans: D) ଦେଶଜ
  10. ‘ଦନ୍ତୀ’ ଶବ୍ଦରେ କେଉଁ ପ୍ରତ୍ୟୟ ଲାଗିଛି?
    A) ଅନ୍
    B) ଈ
    C) ତିନ୍
    D) ଇନ୍
    Ans: D) ଇନ୍
  11. ‘ରୋଗୀ’ ଶବ୍ଦର ବିପରୀତ ଶବ୍ଦ?
    A) ଅରୋଗ
    B) ଅରୋଗୀ
    C) ନିରୋଗ
    D) ନିରୋଗୀ
    Ans: C) ନିରୋଗ
  12. ନିମ୍ନଲିଖିତ ମଧ୍ୟରୁ କେଉଁ ଯୋଡ଼ା ସମପର୍ଯ୍ୟାୟବାଚୀ ଶବ୍ଦ ନୁହଁନ୍ତି?
    A) ସିଂହ-କେଶରୀ
    B) ଦେବତା-ଅମର
    C) ନଭ-ନେତ୍ର
    D) ଗାଭୀ-ଗୋମାତା
    Ans: C) ନଭ-ନେତ୍ର
  13. ନିମ୍ନୋକ୍ତ ମଧ୍ୟରୁ କେଉଁଟି ଉପସର୍ଗରେ ଗଠିତ ଶବ୍ଦ ନୁହେଁ?
    A) ଫିରୋଜ
    B) ପ୍ରକୃଷ୍ଟ
    C) ଉପନୀତ
    D) ପ୍ରଗାଢ
    Ans: A) ଫିରୋଜ
  14. ‘ଇନ୍ଦ୍ର ‘ ଶବ୍ଦର ‘ ନ୍ଦ୍ର’ ଯୁକ୍ତାକ୍ଷରଟି କେଉଁ କେଉଁ ବର୍ଷର ସଂଯୋଗରୁ ସୃଷ୍ଟି ହୋଇଛି?
    A) ନ୍+ଦ୍+ର
    B) ଦ୍+ର+ନ୍
    C) ଦ୍+ନ୍+ର
    D) ନ୍ଦ+ର
    Ans: A) ନ୍+ଦ୍+ର
  15. ‘ଅଗମୂଳ ନଥିବା’ – ରୂଢିଟିର ଅର୍ଥ କ’ଣ?
    A) କିଛି ନ ବୁଝିବା
    B) ଶୃଙ୍ଖଳା ନରହିବା
    C) ଅଗ୍ରାଧିକାର
    D) ସବୁବେଳେ ଗପିବା
    Ans: B) ଶୃଙ୍ଖଳା ନରହିବା
  16. ଶିରଃ+ଛେଦ=?
    A) ଶିରୋଛେଦ
    B) ଶିରଛେଦ
    C) ଶିରଚ୍ଛେଦ
    D) ଶିରଃଛେଦ
    Ans: C) ଶିରଚ୍ଛେଦ
  17. ନିମ୍ନଲିଖିତ ମଧ୍ୟରୁ କେଉଁଟି ‘ପ୍ରାଚୀ’ ଶବ୍ଦର ବିପରୀତ ଶବ୍ଦ?
    A) ପ୍ରତୀଚୀ
    B) ପୂର୍ବ
    C) ଅବାଚୀ
    D) ଉଦିଚୀ
    Ans: A) ପ୍ରତୀଚୀ
  18. ‘କଳାକାଠ ପଡ଼ିଯିବା’– ରୂଢିଟିର ଅର୍ଥ କ’ଣ?
    A) ପୋଡ଼ା କାଠ
    B) କଳାଲୋକ
    C) କଳା ରଙ୍ଗର କାଠ
    D) ଶୁଖିଲା ଦିଶିବା
    Ans: D) ଶୁଖିଲା ଦିଶିବା
  19. ‘ଦେଖିବା ‘ – ଉକ୍ତ କ୍ରିୟାର ସଂସ୍କୃତ ଧାତୁ ନିମ୍ନଲିଖିତ ମଧ୍ୟରୁ କେଉଁଟି ହେବ ସୂଚାଅ?
    A) ଦୃଷ୍ଟ
    B) ଦ୍ରଶ୍
    C) ଦେଖ୍
    D) ଦୃଶ୍
    Ans: C) ଦେଖ୍
  20. ‘ମିଶ୍ର’ – ଶବ୍ଦଟି କେଉଁ ବିଭକ୍ତି ଯୋଗେ ଗଠିତ?
    A) ପଞ୍ଚମୀ
    B) ଦ୍ଵିତୀୟା
    C) ପ୍ରଥମା
    D) ଷଷ୍ଠୀ
    Ans: C) ପ୍ରଥମା
  21. ନିମ୍ନଲିଖିତ ମଧ୍ୟରୁ କେଉଁଟି ଭିନ୍ନ ଶବ୍ଦ ଚିହ୍ନାଅ?
    A) ଆସନ୍ନ
    B) ନିକଟ
    C) ଦୂର
    D) ସମୀପ
    Ans: C) ଦୂର
  22. ‘ଓହ୍ଲାଇବା’ – ଏହି ଅମିଶ୍ର କ୍ରିୟାର ମିଶ୍ରକ୍ରିୟା କଣହେବ?
    A) ଅବତରଣ କରିବା
    B) ତଳକୁ ଯିବା
    C) ପଛକୁ ଫେରିବା
    D) ଓହ୍ଲାଇହେବା
    Ans: A) ଅବତରଣ କରିବା
  23. ନିମ୍ନଲିଖିତ ମଧ୍ୟରୁ ଭିନ୍ନ ପର୍ଯ୍ୟାୟବାଚୀ ଶବ୍ଦଟିକୁ ଚିହ୍ନାଅ?
    A) ମହେନ୍ଦ୍ର
    B) ମଦନ
    C) ମନ୍ମଥ
    D) ଅନଙ୍ଗ
    Ans: A) ମହେନ୍ଦ୍ର
  24. ‘ଫାଙ୍କା ଆୱାଜ’ ରୁଢିଟିର ଅର୍ଥ କ’ଣ?
    A) ଖାଲି ମାଠିଆ
    B) ମିଛ କଥା
    C) ବୃଥା ଧମକ
    D) ଫାଙ୍କା ସ୍ଥାନରେ ଶବ୍ଦ କରିବା
    Ans: C) ବୃଥା ଧମକ
  25. ‘ଅର୍ବାଚୀନ’ ଶବ୍ଦର ବିପରୀତ ଶବ୍ଦ?
    A) ନବୀନ
    B) ନୂତନ
    C) ପ୍ରସୂନ
    D) ପ୍ରାଚୀନ
    Ans: D) ପ୍ରାଚୀନ
  26. ନିମ୍ନଲିଖିତ ମଧ୍ୟରୁ କେଉଁ ଉପସର୍ଗଟିକୁ ଯୋଡ଼ିଦେଲେ ‘ଘାତ’ ଶବ୍ଦଟିର ବିପରୀତ ଅର୍ଥକୁ ବୁଝାଇବ?
    A) ପ୍ର
    B) ପ୍ରତି
    C) ପରା
    D) ଅନୁ
    Ans: B) ପ୍ରତି
  27. ‘ଅମୃତ’ ଶବ୍ଦର ବିପରୀତ ଶବ୍ଦ?
    A) ଉଦକ
    B) ବାରି
    C) ବିଷ
    D) ତୋୟ
    Ans: C) ବିଷ
  28. ‘କାଠିକର ପାଠ’– ରୂଢିଟିର ଅର୍ଥ କ’ଣ?
    A) କାଠି ମାଧ୍ୟମରେ କାର୍ଯ୍ୟ
    B) ବଡ଼ କଠିନ କାମ
    C) ଉତ୍ତମ କାମ
    D) ସହଜ କାମ
    Ans: B) ବଡ଼ କଠିନ କାମ
  29. ‘ପାଣି ଛଡ଼ାଇବା’– ରୂଢିଟିର ଅର୍ଥ କ’ଣ?
    A) ପୂଜା କରିବା
    B) ସମକକ୍ଷ ନହେବା
    C) ଗାଧୋଇବା
    D) ଆଶା ନ ରଖି
    Ans: D) ଆଶା ନ ରଖି
  30. ‘ତିରୋଧାନ’ ଶବ୍ଦର ବିପରୀତ ଶବ୍ଦ?
    A) ଉଦ୍‌ଘାଟନ
    B) ବିସର୍ଜନ
    C) ଆବାହନ
    D) ଆବିର୍ଭାବ
    Ans: D) ଆବିର୍ଭାବ
  31. 1, 6, 11,____ what will be its 15th term?
    (A) 41
    (B) 61
    (C) 71
    (D) 56
    ANS:- (C) 71
    Sol:- To find the pattern and subsequently the 15th term, let’s analyze the differences between consecutive terms:
    The difference between 6 and 1 is 5.
    The difference between 11 and 6 is also 5.
    It appears that the series is increasing by 5 each time. So, to find the 15th term, we can continue this pattern:
    1 = 1st term
    1 + 5 = 6; 2nd term
    6 + 5 = 11; 3rd term
    Continuing this pattern:
    11 + 5 = 16; 4th term
    16 + 5 = 21; 5th term
    21 + 5 = 26; 6th term
    26 + 5 = 31; 7th term
    31 + 5 = 36; 8th term
    36 + 5 = 41; 9th term
    41 + 5 = 46; 10th term
    46 + 5 = 51; 11th term
    51 + 5 = 56; 12th term
    56 + 5 = 61; 13th term
    61 + 5 = 66; 14th term
    66 + 5 = 71; 15th term
    So, the 15th term in the series is 71.
  32. What is the missing number in the series: 3, 7, 12, 18, ?, 33?
    A) 22
    B) 23
    C) 24
    D) 25
    ANS:- D) 25
    Sol:- To find the missing number in the series, let’s analyze the pattern:
    The differences between consecutive numbers in the series are increasing by 1 each time:
    7 – 3 = 4
    12 – 7 = 5
    18 – 12 = 6
    So, following this pattern, the next difference should be 7:
    The next number should be 18 + 7 = 25.
    Next number is 33 – 25 = 8
    Therefore, the missing number in the series is 25.
  33. Complete the analogy:
    Water : Wet:: Fire:?
    Options:
    a) Hot
    b) Cold
    c) Burning
    d) Dry
    d) Dry
    Sol:- Water is wet, and fire is dry. Options (a), (b), and (c) are related to properties or states of fire, but only option (d) represents the opposite property to “wet.”
  34. Complete the analogy:
    Plane: Pilot:: Ship:?
    Options:
    a) Driver
    b) Sailor
    c) Captain
    d) Navigator
    ANS:- c) Captain
    Sol:- A pilot flies a plane, and a captain commands a ship. Options (a), (b), and (d) are roles associated with transportation, but only option (c) corresponds to the person in charge of a ship.
  35. Solve the equation for x: 3x + 6 = 21
    a) x = 3
    b) x = 5
    c) x = 6
    d) x = 7
    NS:- b) x=5
    Sol:- To solve the equation 3x + 6 = 21 for x, you first isolate x by subtracting 6 from both sides of the equation:
    3x + 6 − 6 = 21 − 6
    3x + 0 = 15
    Then, divide both sides by 3 to solve for
    3x/3 = 15/3
    X = 5
    So, the correct answer is b) x = 5
  36. Balance the equation: 25 ÷ 5 + 3 = 4 × 2 – ?
    a) 2
    b) 4
    c) 6
    d) 8
    ANS:- d) 0
    Sol:- To balance the equation 25 ÷ 5 + 3 = 4 × 2 − ?, let’s solve each side separately:
    Left side:
    25 ÷ 5 + 3
    =5 + 3
    =8
    Right side:
    4 × 2 − ?
    = 8 −?
    To balance the equation, we need to find the value of? such that both sides are equal.
    From the left side, 25÷5+3=8, so we need the right side to also equal 8. Therefore, 8−?=8.
    To solve for ? we subtract 8 from both sides:
    8 − ? = 8
    8 − 8
    ? = 0
    ? must be 0 to balance the equation.
    Therefore, the correct answer is d) 0
  37. If ‘V’ represents addition, ‘^’ represents subtraction, ‘X’ represents multiplication, and ‘@’ represents division, what does the expression ’12 V 3 ^ 4 X 2 @ 2′ equal to?
    (A) 11
    (B) 54
    (C) 36
    (D) 30
    ANS:- (A) 11
    Sol:- o solve the expression using the BODMAS (Brackets, Orders (i.e., powers and square roots, etc.), Division and Multiplication, Addition and Subtraction) theory:
    12 V 3 ^ 4 X 2 @ 2
    Given:
    V represents addition (+)
    ^ represents subtraction (-)
    X represents multiplication (*)
    @ represents division (/)
    So the expression becomes: 12 + 3 – 4 * 2 / 2
    Let’s break down the expression step by step:
    First, we perform the multiplication and division operations:
    2 / 2 = 1
    Now, the expression becomes: 12 + 3 – 4 * 1 = 12 + 3 – 4
    Next, we perform the addition operation (V):
    12 + 3 = 15
    Now, the expression becomes:
    15 – 4
    Next, we perform the subtraction (-) operation:
    15 – 4 = 11
    So, the expression ’12 V 3 ^ 4 X 2 @ 2′ equals 11.
  38. If 5 workers can complete a task in 12 days, how many days would it take for 8 workers to complete the same task?
    (A) 5
    (B) 8
    (C) 7.5
    (D) 9
    ANS:- (C) 7.5
    Sol:- To solve this problem, we can use the concept of man-days, which is the amount of work done by one person in one day.
    Given:
    5 workers can complete the task in 12 days.
    So, the total man-days required to complete the task are:
    5 workers * 12 days = 60 man-days
    Now, if 8 workers are to complete the same task, you can find out how many days it would take by dividing the total man-days required by the number of workers:
    Number of days = Total man-days / Number of workers
    Number of days = 60 man-days / 8 workers
    Number of days = 7.5 days
    So, it would take approximately 7.5 days for 8 workers to complete the same task.
  39. Which of the following is a valid conclusion from the statements:
    Statements:
    (A) All cats are mammals.
    (B) Some mammals are dogs.
    Conclusion:
    I. Some cats are dogs.
    II. Some dogs are cats.
    III. All dogs are mammals.
    IV. Some mammals are cats.
    Options:-
    (a) None follows
    (b) Only IV follows
    (c) Only I follow
    (d) Both I and IV follow
    ANS:- (c) Only I follow.
    Sol:- Let’s analyze each conclusion:
    Conclusion I: Some cats are dogs.
    This conclusion logically follows from the given statements. If all cats are mammals (statement A) and some mammals are dogs (statement B), then it’s logical to conclude that some cats are dogs.
    Conclusion II: Some dogs are cats.
    This conclusion cannot be logically derived from the given statements. While some mammals are dogs and all cats are mammals, it doesn’t necessarily mean that some dogs are cats.
    Conclusion III: All dogs are mammals.
    This conclusion cannot be derived from the given statements. While all cats are mammals and some mammals are dogs, it doesn’t mean that all dogs are mammals. Statement A explicitly states that all cats are mammals, but it doesn’t extend to all dogs. Conclusion IV: Some mammals are cats.
    This conclusion cannot be logically inferred from the given statements. Although all cats are mammals and some mammals are dogs, it doesn’t imply that some mammals are cats.
    Based on the analysis:
    Conclusion I “Some cats are dogs” is valid.
    Conclusion IV “Some mammals are cats” cannot be logically inferred from the given statements.
    The other conclusions (II and III) are not valid.
    So, the correct answer is (c) Only I follow.
  40. If “All fish are aquatic” and “Some aquatic are mammals” are true, which of the following conclusions follows logically?
    Conclusion:
    I. Some mammals are fish.
    II. All mammals are fish.
    III. Some fish are aquatic.
    IV. All aquatic are fish.
    Options:-
    (a) None follows
    (b) Only IV follows
    (c) Only I follow
    (d) Both I and IV follow
    ANS:- (c) Only I follow.
    Sol:- Let’s analyze each conclusion:
    Conclusion I: Some mammals are fish.
    This conclusion logically follows from the given statements. If all fish are aquatic and some aquatic beings are mammals, then it’s logical to conclude that some mammals are fish.
    Conclusion II: All mammals are fish.
    This conclusion cannot be logically inferred from the given statements. While some aquatic beings are mammals and all fish are aquatic, it doesn’t necessarily mean that all mammals are fish. There may be other types of mammals that are not fish.
    Conclusion III: Some fish are aquatic.
    This conclusion is redundant and doesn’t add any new information. It is already given in the statements that all fish are aquatic, so this conclusion is not necessary.
    Conclusion IV: All aquatic are fish.
    This conclusion cannot be logically inferred from the given statements. While all fish are aquatic, it doesn’t imply that all aquatic beings are fish. Some aquatic beings may be mammals, as stated in the premises.
    Based on the analysis:
    Conclusion I “Some mammals are fish” is valid.
    Conclusion IV “All aquatic are fish” is not valid.
    The other conclusions (II and III) are not necessary or valid.
    So, the correct answer is (c) Only I follow.
  41. Shanu is facing south, She turns 90 degrees left, then she turns 90 degrees right, then she turns 180 degrees left. Now in which direction is she facing?
    A) North
    B) East
    C) South
    D) West
    ANS:- A) North
    Sol:- Let’s analyze Shanu’s movements:
    Shanu starts facing South.
    She turns 90 degrees left, which means she is now facing East.
    Then she turns 90 degrees right from East, which means she is now facing South again.
    Next, she turns 180 degrees left from South, which means she makes a complete turn and ends up facing North.
    So, after all the movements, Shanu is facing North.
    Therefore, Shanu is facing A) North
  42. Which of the following statements is true about acceleration?
    (A) Acceleration is always positive.
    (B) Acceleration is the rate of change of speed.
    (C) Acceleration can be positive, negative, or zero.
    (D) Acceleration is a scalar quantity.
    ANS:- The correct answer is (C) Acceleration can be positive, negative, or zero.
    Sol:- Acceleration is defined as the rate of change of velocity concerning time. It can be positive if an object is speeding up, negative if it’s slowing down, or zero if it’s moving at a constant velocity.
    It’s essential to understand that acceleration is a vector quantity because it has both magnitude and direction, unlike a scalar quantity which only has magnitude.
    While option (B) mentions the rate of change of speed, acceleration is the rate of change of velocity, which includes both speed and direction.
    Option (A) is incorrect because acceleration can be negative if the object is decelerating or slowing down.
    Option (D) is incorrect because acceleration is a vector quantity, not a scalar quantity.
    The correct answer is (C) Acceleration can be positive, negative, or zero.
  43. What does a negative velocity indicate?
    (A) Acceleration
    (B) Deceleration
    (C) Motion in the opposite direction
    (D) No motion
    ANS:- (C) Motion in the opposite direction
    Sol:- Velocity is a vector quantity that describes the rate of change of an object’s position concerning time, including both speed and direction.
    A negative velocity indicates that the object is moving in the opposite direction to the positive direction defined in the coordinate system.
    Option (A) Acceleration refers to the rate of change of velocity, not velocity itself.
    Option (B) Deceleration refers to a decrease in velocity, but it does not specify the direction of motion.
    Option (D) No motion would be indicated by a velocity of zero, not a negative velocity.
    The correct answer is (C) Motion in the opposite direction
  44. Gita said to Mamta, “The girl I met yesterday at the beach was the youngest daughter of my friend’s mother’s brother-in-law.” How is the girl related to Gita’s friend?
    (A) Cousin
    (B) Daughter
    (C) Friend
    (D) Aunt
    ANS:- (A) Cousin
    Sol:- Let’s restructure the statement to make it clearer:
    Gita said to Mamta, “The girl I met yesterday at the beach was the youngest daughter of my friend’s mother’s brother-in-law.”
    Now, let’s break it down:
    Gita’s friend’s mother’s brother-in-law is Gita’s friend’s uncle.
    The youngest daughter of Gita’s friend’s uncle is Gita’s friend’s cousin.
    So, the girl is related to Gita’s friend as her cousin.
  45. Manisha said, “This girl is the wife of the grandson of my mother.” How is Manisha related to the girl?
    (A) Brother
    (B) Grandfather
    (C) Husband
    (D) Father-in-law
    ANS:- (D) Father-in-law
    Sol:- Manisha said, “This girl is the wife of the grandson of my mother.”
    Let’s break this down:
    Manishas mother’s grandson is Manisha’s nephew.
    The wife of Manisha’s nephew is Manisha’s nephew’s wife.
    Manisha’s nephew’s wife is Manisha’s nephew’s daughter-in-law.
    So, Manisha is related to the girl as her father-in-law, which corresponds to option (D).
  46. Which of the following is the correct representation of the number five in binary?
    A) 101
    B) 100
    C) 110
    D) 111
    ANS:- B) 101
    Sol:- In binary, the number 5 is represented as 101, which corresponds to
    = 1 x 22 + 0 x 21 + 1 x 20
    = [1 x (2 x 2)] + [0 x (2 x 1)] + [1 x 20]
    = (1 x 4) + (0 x 2) + (1)
    = 4 + 0 + 1
    = 5
  47. Which of the following is a prime number?
    A) 20
    B) 17
    C) 16
    D) 15
    ANS:- B) 17
    Sol:- A prime number is a positive integer greater than 1 that has no positive divisors other than 1 and itself.
    Let’s check each option:
    A) 20 – This is not a prime number because it has divisors other than 1 and itself (2, 4, 5, 10).
    B) 17 – This is a prime number because its only divisors are 1 and 17.
    C) 16 – This is not a prime number because it has divisors other than 1 and itself (2, 4, 8).
    D) 15 – This is not a prime number because it has divisors other than 1 and itself (3, 5).
    So, the correct answer is B) 17.
  48. Find the domain of the function f(x) = √(x+1).
    A) {x ∈ ℝ : x ≥ -1}
    B) {x ∈ ℝ : x ≤ -1}
    C) {x ∈ ℝ : x > -1}
    D) {x ∈ ℝ : x < -1}
    S2. ANS:- A) {x ∈ ℝ : x ≥ -1}
    Sol:- The domain of a square root function includes all real numbers that make the expression under the square root non-negative.
    For the function f(x) = √x+1, the expression under the square root, x + 1, must be greater than or equal to zero for the function to be defined.
    So, we solve the inequality x + 1 ≥ 0 for X
    x + 1 ≥ 0
    x ≥ −1
    So, the domain of the function is all real numbers greater than or equal to -1.
    The correct answer is A) {x ∈ ℝ : x ≥ -1}
  49. Determine if the function f(x) = x22 is one-to-one.
    A) Yes
    B) No
    S3. ANS:- B) No
    Sol:-The function f(x)=x2 is not one-to-one.
    A function is one-to-one (or injective) if each element of the function’s codomain is mapped to by at most one element of its domain.
    However, in the case of f(x) = x2, multiple inputs can result in the same output. For example, both f(2) = 4 and f(−2) = 4, indicating that the function fails the horizontal line test.
    So, the correct answer is B) No
  50. What is the solution to the equation 5x + 3 = 28?
    A) x = 5
    B) x = 6
    C) x = 7
    D) x = 8
    ANS:- A) x=5
    Sol:- To solve the equation 5x + 3 = 28, we need to isolate x on one side of the equation.
    First, subtract 3 from both sides:5x+3−3=28−3
    5x = 25
    Then, divide both sides by 5 to solve for
    5x/5 = 25/5
    ​x=5
    So, the correct answer is A) x=5
  51. Which of the following is an example of a linear equation in three variables?
    A) x+2y=3z
    B) x2+y2 =9
    C) 2x+3y−z=7
    D) x+y+z=1
    ANS:- A) x+2y=3z
    Sol:- Option A) x+2y=3z is indeed an example of a linear equation in three variables. It includes all three variables (x, y, z) raised to the first power, satisfying the definition of a linear equation in three variables.
  52. What does the “±” sign represent in the quadratic formula?
    A) The discriminant
    B) The coefficient of x
    C) The presence of two solutions
    D) The constant term
    ANS:- C) The presence of two solutions
    Sol:- In the quadratic formula  Weekly Practice Quiz For For OSSSC RI,ARI, Amin: June-8_3.1
    , the “±” sign indicates that there are two possible solutions for the quadratic equation. This is because the quadratic formula considers both the positive and negative roots of the discriminant, resulting in two solutions. Therefore, option C) The presence of two solutions
    is the correct choice.
  53. Calculate log10 (85) using antilogarithm.
    A) 1.928
    B) 1.894
    C) 1.956
    D) 1.863
    ANS:- A) 1.928
    Sol:- To calculate log10(85) using antilogarithm, you’ll want to find the number that 10 needs to be raised to equal 85.
    So, using the antilogarithm:
    log10(85) = x
    10^x = 85
    Now, solving for x:
    x ≈ log10(85) = 1.929
    The closest option is A) 1.928.
  54. If antilog(2.876)=768, what is log10 (768)?
    A) 2.876
    B) 3.876
    C) 4.876
    D) 5.876
    ANS:- A) 2.876.
    Sol:- To find log10 (768), we can use the fact that antilog(2.876) = 768.
    Antilog(2.876) = 768 means 10^2.876 = 768.
    So, log10 (768) = 2.876.
    Therefore, the correct answer is A) 2.876.
  55. How many doublings are required to reach a value of 32 with a base 2 logarithm?
    A) 2
    B) 3
    C) 4
    D) 5
    ANS:- D) 5.
    Sol:- To find out how many doublings are required to reach a value of 32 with a base 2 logarithm, we need to determine the exponent to which 2 must be raised to get 32.
    In other words, we need to solve the equation
    2x =32
    x = log2 (32)
    x = log2 (25)
    x = 5
    So, 5 doublings are required to reach a value of 32 with a base 2 logarithm.
    Therefore, the correct answer is D) 5.
  56. What is the approximate value of the base e?
    A) 3.14159
    B) 2.7183
    C) 10
    D) 1.4142
    NS:- B) 2.7183
    Sol:- The base e, also known as Euler’s number, is approximately equal to 2.7183. So the correct answer is B) 2.7183
  57. What is the value of secant θ if cosine θ = 0.6?
    A) 0.6
    B) 1.67
    C) 1.25
    D) 1.67
    ANS:- D) 1.67.
    Sol:- To find the value of secant θ, we can use the identity:
    sec(θ) = 1 / cos(θ)
    ​Given that cos(θ) = 0.6, we can plug this value into the formula:
    sec(θ) = 1/ 0.6
    ​ = 5 / 3
    = 1.67.
    So, the value of secant θ is option D) 1.67.
  58. If cotθ= 4 / 3, what is the value of tanθ?
    A) 3 / 4
    B) 4 / 3
    C) 9 / 16
    D) 16 / 9
    ANS:- A) 3/4
    Sol:- To find the value of tanθ when cotθ = 4 / 3, we can use the relationship between cotangent and tangent:
    Cotθ = 1 / tanθ
    Given that cotθ = 4/3, we can rewrite this equation as:
    4/3 = 1/tanθ
  59. A flagpole casts a shadow of 10 meters when the angle of elevation of the sun is 45°. What is the height of the flagpole?
    a) 5 √2 meters
    b) 10 meters
    c) 10 √2 meters
    d) 20 meters
    ANS:- 10 meters
    Sol:- To find the height of the flagpole, we can use the trigonometric relationship between the height of the flagpole, the length of its shadow, and the angle of elevation of the sun.
    Let h be the height of the flagpole, and s be the length of its shadow.
    Given that the angle of elevation of the sun is 45∘, we can use the tangent function:
    tan(45∘) = h/ s
    ​Since tan(45∘) = 1, we have:
    1 = h/ 10
    Multiplying both sides by 10, we get:
    h = 10
    So, the height of the flagpole is 10 meters, which matches option b) 10 meters
  60. A kite is flying at an angle of elevation of 45°. If the length of the kite string is 50 meters, how high is the kite above the ground?
    a) 25 √2 meters
    b) 50 meters
    c) 50 √2 meters
    d) 100 meters
    ANS:- a) 25 √2 meters,
    Sol:- We can use trigonometry to find the height of the kite above the ground.
    Let h be the height of the kite above the ground.
    Given that the angle of elevation of the kite is 45∘, and the length of the kite string (hypotenuse) is 50 meters, we can use the sine function:
    sin(45∘) = h/ 50
    Since ​sin(45∘) = √2 /2, we have:
    √2 /2 = h/ 50
    ​Multiplying both sides by 50, we get:
    h = √2/ 2 × 50
    h = 25√2
    So, the height of the kite above the ground is 25 √2 meters,
  61. Who was the first Governor-General of independent India?
    a) Lord Mountbatten
    b) Jawaharlal Nehru
    c) C. Rajagopalachari
    d) Lord Louis Mountbatten
    Answer: d) Lord Louis Mountbatten
  62. Who was the founder of the Maurya Empire?
    a) Chandragupta Maurya
    b) Ashoka
    c) Bindusara
    d) Bimbisara
    Answer: a) Chandragupta Maurya
  63. The Battle of Plassey was fought in which year?
    a) 1757
    b) 1761
    c) 1773
    d) 1789
    Answer: a) 1757
  64. Who was the first Indian woman president of the Indian National Congress?
    a) Sarojini Naidu
    b) Annie Besant
    c) Vijaya Lakshmi Pandit
    d) Sucheta Kriplani
    Answer: a) Sarojini Naidu
  65. The Indus Valley Civilization was primarily located in which modern-day country?
    a) India
    b) Pakistan
    c) Bangladesh
    d) Nepal
    Answer: b) Pakistan
  66. The Vedic Period in Indian history is characterized by the composition of which texts?
    a) Vedas
    b) Upanishads
    c) Ramayana
    d) Mahabharata
    Answer: a) Vedas
  67. Who was the author of the book “Discovery of India”?
    a) Jawaharlal Nehru
    b) Mahatma Gandhi
    c) Rabindranath Tagore
    d) Sardar Vallabhbhai Patel
    Answer: a) Jawaharlal Nehru
  68. The first War of Indian Independence, also known as the Sepoy Mutiny, took place in which year?
    a) 1857
    b) 1858
    c) 1865
    d) 1875
    Answer: a) 1857
  69. Who was the last Mughal emperor of India?
    a) Bahadur Shah II
    b) Shah Jahan
    c) Aurangzeb
    d) Humayun
    Answer: a) Bahadur Shah II
  70. The Quit India Movement was launched in which year?
    a) 1942
    b) 1930
    c) 1947
    d) 1920
    Answer: a) 1942
  71. The founder of the Gupta Dynasty was?
    a) Chandragupta II
    b) Chandragupta I
    c) Samudragupta
    d) Skandagupta
    Answer: b) Chandragupta I
  72. Who was the first Indian woman to become the President of the United Nations General Assembly?
    a) Sarojini Naidu
    b) Vijaya Lakshmi Pandit
    c) Indira Gandhi
    d) Jayaprakash Narayan
    Answer: b) Vijaya Lakshmi Pandit
  73. The Indian National Congress was founded in which year?
    a) 1885
    b) 1905
    c) 1947
    d) 1920
    Answer: a) 1885
  74. Who among the following was the first Muslim President of the Indian National Congress?
    a) Badruddin Tyabji
    b) Syed Ahmed Khan
    c) Maulana Abul Kalam Azad
    d) Rafi Ahmed Kidwai
    Answer: a) Badruddin Tyabji
  75. The Battle of Buxar was fought in which year?
    a) 1764
    b) 1761
    c) 1757
    d) 1770
    Answer: a) 1764
  76. Who was the first British Governor-General of India?
    a) Lord Cornwallis
    b) Lord Warren Hastings
    c) Lord William Bentinck
    d) Lord Dalhousie
    Answer: c) Lord William Bentinck
  77. The Aryans entered India from which direction?
    a) North-West
    b) North-East
    c) South-West
    d) South-East
    Answer: a) North-West
  78. The doctrine of Swadeshi was popularized during which movement?
    a) Non-Cooperation Movement
    b) Quit India Movement
    c) Khilafat Movement
    d) Civil Disobedience Movement
    Answer: a) Non-Cooperation Movement
  79. Who authored the famous book ‘Anand Math’?
    a) Rabindranath Tagore
    b) Bankim Chandra Chattopadhyay
    c) Ishwar Chandra Vidyasagar
    d) Raja Ram Mohan Roy
    Answer: b) Bankim Chandra Chattopadhyay
  80. Who is known as the Father of Indian Renaissance?
    a) Raja Ram Mohan Roy
    b) Swami Vivekananda
    c) Mahatma Gandhi
    d) Jawaharlal Nehru
    Answer: a) Raja Ram Mohan Roy
  81. The Indian National Army (INA) was formed by which leader?
    a) Mohan Singh
    b) Bhagat Singh
    c) Chandrashekhar Azad
    d) Lala Lajpat Rai
    Answer: a) Mohan Singh
  82. Who was the first Governor-General of Bengal?
    a) Warren Hastings
    b) Lord Clive
    c) Robert Clive
    d) Lord William Bentinck
    Answer: a) Warren Hastings
  83. The Battle of Panipat between Babur and Ibrahim Lodi was fought in which year?
    a) 1526
    b) 1556
    c) 1565
    d) 1605
    Answer: a) 1526
  84. The Swaraj Party was formed by which leaders after the withdrawal of the Non-Cooperation Movement?
    a) Motilal Nehru and Chittaranjan Das
    b) Subhas Chandra Bose and Jawaharlal Nehru
    c) Bhagat Singh and Chandrashekhar Azad
    d) Mahatma Gandhi and Sardar Vallabhbhai Patel
    Answer: a) Motilal Nehru and Chittaranjan Das
  85. The Indian National Congress split into two factions, the Extremists and the Moderates, during which session?
    a) Surat Session (1907)
    b) Lucknow Session (1916)
    c) Calcutta Session (1906)
    d) Lahore Session (1929)
    Answer: a) Surat Session (1907)
  86. Who is considered the architect of the Indian Constitution?
    a) Dr. B.R. Ambedkar
    b) Jawaharlal Nehru
    c) Mahatma Gandhi
    d) Sardar Vallabhbhai Patel
    Answer: a) Dr. B.R. Ambedkar
  87. The Khilafat Movement was launched to protest against the dismantling of the Ottoman Caliphate after World War I. Who were the two leaders primarily associated with this movement?
    a) Muhammad Ali and Shaukat Ali
    b) Maulana Abul Kalam Azad and Hakim Ajmal Khan
    c) Motilal Nehru and Chittaranjan Das
    d) Gopal Krishna Gokhale and Bal Gangadhar Tilak
    Answer: a) Muhammad Ali and Shaukat Ali
  88. Who was the first Indian woman to climb Mount Everest?
    a) Bachendri Pal
    b) Arunima Sinha
    c) Premlata Agarwal
    d) Santosh Yadav
    Answer: a) Bachendri Pal
  89. The Jallianwala Bagh massacre took place in which year?
    a) 1919
    b) 1920
    c) 1921
    d) 1918
    Answer: a) 1919
  90. Who is known as the Iron Man of India?
    a) Sardar Vallabhbhai Patel
    b) Subhas Chandra Bose
    c) Bhagat Singh
    d) Jawaharlal Nehru
    Answer: a) Sardar Vallabhbhai Patel
  91. Who was honored with the KISS Humanitarian Award in 2021?
    A) Achyuta Samanta
    B) Ratan Tata
    C) Binod Kumar Pasayat
    D) Guru Gopinath Swain
    Answer: B) Ratan Tata
  92. Who initiated the KISS Humanitarian Award?
    A) Ratan Tata
    B) Achyuta Samanta
    C) Guru Gopinath Swain
    D) Binod Kumar Pasayat
    Answer: B) Achyuta Samanta
  93. What does the KISS Humanitarian Award trophy symbolize?
    A) Wealth and prosperity
    B) Power and influence
    C) Selfless service to humanity
    D) Artistic excellence
    Answer: C) Selfless service to humanity
  94. What does the single pair of hands holding the heart symbolize in the KISS Humanitarian Award trophy?
    A) Compassionate service to humanity
    B) Personal achievement
    C) Cultural diversity
    D) Technological innovation
    Answer: A) Compassionate service to humanity
  95. What significant social message does the KISS Humanitarian Award trophy convey?
    A) Pursuit of wealth and power
    B) Advancement of technology
    C) Empowerment and enlightenment of individuals
    D) Political activism
    Answer: C) Empowerment and enlightenment of individuals
  96. Who among the following received the Padma Shri award for contributions to Sambalpuri/Koshli literature?
    A) Ratan Tata
    B) Binod Kumar Pasayat
    C) Guru Gopinath Swain
    D) Achyuta Samanta
    Answer: B) Binod Kumar Pasayat
  97. Which award did Shri Binod Kumar Pasayat receive in 2008?
    A) Padma Shri
    B) Sarala Puraskar
    C) Odisha Sahitya Academy Puraskar
    D) Odisha Sangeet Natak Akademi award
    Answer: B) Sarala Puraskar
  98. What is Guru Gopinath Swain known for?
    A) Traditional barbering
    B) Traditional Krishna Leela
    C) Modern dance forms
    D) Sculpture art
    Answer: B) Traditional Krishna Leela
  99. At what age did Guru Gopinath Swain start learning Krishna Leela?
    A) 5
    B) 10
    C) 15
    D) 20
    Answer: B) 10
  100. What language is the Krishna Leela authored by Babaji Dukhisyama Dasa?
    A) Sanskrit
    B) Hindi
    C) Odia
    D) Bengali
    Answer: C) Odia
  101. When were Electronic Voting Machines (EVMs) first used in India?
    a) 1984
    b) 1982
    c) 1992
    d) 2002
    Answer: b) 1982
  102. What is the primary purpose of Electronic Voting Machines (EVMs)?
    a) Enhancing voter turnout
    b) Streamlining the voting process
    c) Reducing election costs
    d) Ensuring secrecy of votes
    Answer: b) Streamlining the voting process
  103. Which component of the Electronic Voting Machine (EVM) is operated by election officials?
    a) Balloting unit
    b) Voter unit
    c) Control unit
    d) Tallying unit
    Answer: c) Control unit
  104. What is the purpose of Voter-Verified Paper Audit Trail (VVPAT)?
    a) To provide a physical printout of the voter’s ballot
    b) To record votes electronically
    c) To verify candidates’ qualifications
    d) To conduct opinion polls
    Answer: a) To provide a physical printout of the voter’s ballot
  105. When was Voter-Verified Paper Audit Trail (VVPAT) first introduced in India?
    a) 2009
    b) 2014
    c) 2019
    d) 2020
    Answer: b) 2014
  106. In which constituency did a special polling team trek 18 kilometers inside a forest to record a single home vote?
    a) Parur Assembly constituency
    b) Idukki Lok Sabha constituency
    c) Munnar forest division
    d) Nooradi tribal settlement
    Answer: d) Nooradi tribal settlement
  107. Who are the primary targets of the “Vote-from-home” exercise?
    a) Voters below 30 years of age
    b) Pregnant women
    c) Elderly voters above 80 years of age and persons with disabilities
    d) Election officials
    Answer: c) Elderly voters above 80 years of age and persons with disabilities
  108. Is participation in the “Vote-from-home” exercise mandatory?
    a) Yes
    b) No, it is voluntary
    c) Only for certain age groups
    d) Only for persons with disabilities
    Answer: b) No, it is voluntary
  109. What form must voters fill out to apply for the “Vote-from-home” facility?
    a) Form 10A
    b) Form 7C
    c) Form 12D
    d) Form 8B
    Answer: c) Form 12D
  110. How many randomly selected polling booths undergo EVM-VVPAT cross-verification in a constituency?
    a) 10
    b) 5
    c) 20
    d) 100
    Answer: b) 5
  111. What caused the rapid spread of the fire in the Sulthan Bathery forest range?
    A) Heavy rainfall
    B) Dried bamboo pods
    C) Low temperature
    D) Thick fog
    Answer: B) Dried bamboo pods
  112. Which factor contributed to the high temperature facilitating the fire’s spread?
    A) Winter rain
    B) Absence of summer rain
    C) Snowfall
    D) Coastal breeze
    Answer: B) Absence of summer rain
  113. Which of the following is NOT a factor contributing to the rapid spread of the fire?
    A) Strong wind
    B) Withered undergrowth
    C) Dense fog
    D) Dried bamboo pods
    Answer: C) Dense fog
  114. What was the geographical location of the Sulthan Bathery forest range?
    A) Karnataka
    B) Kerala
    C) Tamil Nadu
    D) Andhra Pradesh
    Answer: B) Kerala
  115. How did the fire affect the nearby rubber plantation?
    A) Enhanced rubber production
    B) No impact on the plantation
    C) Damaged the plantation
    D) Improved soil fertility
    Answer: C) Damaged the plantation
  116. Which river flows through the Wayanad Wildlife Sanctuary?
    A) Ganges
    B) Yamuna
    C) Kabini
    D) Godavari
    Answer: C) Kabini
  117. What is the primary objective of the Wayanad Wildlife Sanctuary?
    A) Promoting tourism
    B) Conserving biological heritage
    C) Industrial development
    D) Agricultural expansion
    Answer: B) Conserving biological heritage
  118. Which species is NOT found in the Wayanad Wildlife Sanctuary?
    A) Indian elephant
    B) Bengal tiger
    C) Polar bear
    D) Gaur
    Answer: C) Polar bear
  119. What threat does the sanctuary face from human activities?
    A) Excessive tree planting
    B) Illegal poaching
    C) Controlled burning
    D) Environmental conservation
    Answer: B) Illegal poaching
  120. What rare phenomena have been observed at Mount Etna?
    A) Snowstorms
    B) Earthquakes
    C) Volcanic vortex rings
    D) Tornadoes
    Answer: C) Volcanic vortex rings
  121. What is the purpose of a fathometer?
    A) Measuring earthquakes
    B) Measuring rainfall
    C) Measuring ocean depth
    D) Measuring sound intensity
    Answer: C) Measuring ocean depthExplanation: A fathometer is a depth finder that uses sound waves to determine the depth of water.
  122. What is Epsom, England, associated with?
    A) Snooker
    B) Shooting
    C) Polo
    D) Horse racingAnswer: D) Horse racingExplanation: Epsom is a town in Surrey, England, known for Epsom Downs Racecourse,
  123. where The Derby horse race is held.
    Who was the fastest shorthand writer?
    A) Dr. G. D. Bist
    B) J.R.D. Tata
    C) J.M. Tagore
    D) Khudada Khan
    Answer: A) Dr. G. D. BistExplanation: Dr. G. D. Bist, a Guinness Record Holder, achieved a speed of 250 w.p.m. in shorthand.
  124. Which country does golf player Vijay Singh belong to?
    A) USA
    B) Fiji
    C) India
    D) UK
    Answer: B) FijiExplanation: Vijay Singh, known as “The Big Fijian,” is an Indo-Fijian golfer born in Fiji.
  125. “One People, One State, One leader” was the policy of
    A) Stalin
    B) Hitler
    C) Lenin
    D) Mussolini
    Answer: B) HitlerExplanation: This policy was associated with Hitler’s regime in Germany.
  126. What is the full form of IG in the police department?
    Answer: IG stands for Inspector General of Police.
  127. What does DRDL stand for?
    A) Defence Research and Development Laboratory
    B) Department of Research and Development Laboratory
    C) Differential Research and Documentation Laboratory
    D) None of the above
    Answer: A) Defence Research and Development Laboratory
  128. Why does exposure to sunlight help improve health?
    A) The infrared light kills bacteria in the body
    B) Resistance power increases
    C) The pigment cells in the skin get stimulated and produce a healthy tan
    D) The ultraviolet rays convert skin oil into Vitamin D
    Answer: D) The ultraviolet rays convert skin oil into Vitamin D
  129. Where is the headquarters of the registered voluntary association “Transparency International” located?
    (a) Helsinki, Finland
    (b) Geneva, Switzerland
    (c) Berlin, Germany
    (d) Paris, France
    Ans: (c) Berlin, Germany
  130. Who appoints the Secretary-General of the United Nations Organization?
    (a) General Assembly
    (b) Security Council
    (c) Trusteeship Council
    (d) World Bank
    Ans: (a) General Assembly
  131. Which countries does the International Criminal Court lack jurisdiction over?
    (a) France, China, Pakistan, Afghanistan
    (b) UK, France, China, Pakistan
    (c) USA, UK, Russia, France
    (d) USA, Russia, China, Israel
    Ans: (d) USA, Russia, China, Israel
  132. Seasonal variation in Earth’s weather condition is the effect of _____ of the Earth?
    (a) Diastrophism
    (b) Erosion
    (c) Revolution
    (d) Rotation
    Ans: (c) Revolution
  133. Which planet of the Solar system experiences Sunrise on the West?
    (a) Jupiter
    (b) Venus
    (c) Mercury
    (d) Mars
    Ans: (b) Venus
  134. What is the difference between a Nuclear reactor and an atomic bomb?
    (a) No chain reaction takes place in the atomic bomb while it takes place in a nuclear reactor
    (b) The chain reaction in a nuclear reactor is not controlled
    (c) The chain reaction in a nuclear reactor is controlled
    (d) No chain reaction takes place in a nuclear reactor while in the atomic bomb there is a chain reaction
    Ans: (c) The chain reaction in a nuclear reactor is controlled
  135. Which of the following is used for indigestion?
    (a) Baking Soda
    (b) Milk of Magnesia
    (c) Quick lime (calcium oxide)
    (d) All of the above
    Ans: (d) All of the above
  136. What is the process of drilling small-sized holes deep into the Earth’s surface for injecting water, sand, and chemicals in order to obtain shale gas and oil reserves known as?
    (a) Fracking
    (b) Chroning
    (c) Dreecking
    (d) Pulverising
    Ans: (a) Fracking
  137. In superconductivity, the conductivity of a material becomes_______
    (a) Infinite
    (b) Finite
    (c) Zero
    (d) None of the above
    Ans: (a) Infinite
  138. What is the general pitch of the voice of women?
    (a) The same as that of men
    (b) Much lower than that of men
    (c) Higher than that of men
    (d) Marginally lower than that of men
    Ans: (c) Higher than that of men
  139. What is responsible for the diffusion of light in the atmosphere?
    (a) Water vapors
    (b) Helium
    (c) Dust particles
    (d) Carbon Dioxide
    Ans: (c) Dust particles
  140. What source provides some heat to the atmosphere even after sunset?
    (a) Albedo effect
    (b) Latent heat
    (c) Invisible solar radiation
    (d) Terrestrial radiation
    Ans: (d) Terrestrial radiation
  141. The permanent hardness of water is primarily due to the presence of _______.
    (a) Bicarbonates of magnesium and calcium
    (b) Carbonates of sodium and magnesium
    (c) Sulphates of magnesium and calcium
    (d) Sulphates of sodium and potassium
    Ans: (c) Sulphates of magnesium and calcium
  142. Among the following planets, which one does not orbit the Sun from west to east?
    (a) Venus
    (b) Jupiter
    (c) Mars
    (d) Mercury
    Ans: (a) Venus
  143. Which planet(s) does not have a natural satellite?
    (a) Mercury
    (b) Venus
    (c) Both (a) and (b)
    (d) Saturn
    Ans: (c) Both (a) and (b)
  144. Which continent is known as a hollow continent due to its low population in the central areas?
    (a) Europe
    (b) South America
    (c) Australia
    (d) Africa
    Ans: (b) South America
  145. The Murray-Darling system, one of the greatest rivers in the world, is located in which country?
    (a) Germany
    (b) Australia
    (c) Russia
    (d) Canada
    Ans: (b) Australia
  146. Polar Stratospheric Clouds are associated with which environmental phenomenon?
    (a) Artificial Rain
    (b) Acid Rain
    (c) Greenhouse effect
    (d) Ozone layer depletion
    Ans: (d) Ozone layer depletion
  147. Which type of Laser is commonly used in Laser Printers?
    (a) Gas Laser
    (b) Dye Laser
    (c) Excimer Laser
    (d) Semiconductor Laser
    Ans: (d) Semiconductor Laser
  148. The ozone layer prevents the entry of which types of rays into the atmosphere?
    (a) UV-A only
    (b) UV-B only
    (c) UV-C only
    (d) Both UV-B and UV-C
    Ans: (d) Both UV-B and UV-C
  149. What happens to the value of ‘g’ as we move from the equator to the poles?
    (a) Remains the same
    (b) Increases
    (c) Decreases
    (d) None of these
    Ans: (c) Decreases
  150. Which of the following is not considered a structural reform in India?
    (a) Reduction of interest rates
    (b) Land reforms
    (c) Tax reforms
    (d) Delicensing
    Ans: (c) Tax reforms
  151. In which city was Asia’s first dedicated command and control centre for space domain awareness inaugurated?
    A) Mumbai
    B) Delhi
    C) Bengaluru
    D) Hyderabad
    Answer: C) Bengaluru
  152. What type of facility was inaugurated in Bengaluru, making it the first of its kind in Asia?
    A) Hospital
    B) School
    C) Command and control centre for space domain awareness
    D) Research laboratory
    Answer: C) Command and control centre for space domain awareness
  153. Which company’s global headquarters houses the newly opened command and control centre for space domain awareness in Bengaluru?
    A) SpaceX
    B) NASA
    C) Digantara
    D) ISRO
    Answer: C) Digantara
  154. What is the specialization of Digantara, the company that inaugurated the command and control centre in Bengaluru?
    A) Health awareness
    B) Space situational awareness (SSA)
    C) Agricultural technology
    D) Financial services
    Answer: B) Space situational awareness (SSA)
  155. At which meeting did ISRO Chairman S. Somanath announce India’s commitment to achieving debris-free space missions by 2030?
    A) 40th IADC meeting
    B) 42nd IADC meeting
    C) 44th IADC meeting
    D) 46th IADC meeting
    Answer: B) 42nd IADC meeting
  156. What is India’s target year to achieve debris-free space missions, as announced by ISRO Chairman S. Somanath?
    A) 2025
    B) 2030
    C) 2040
    D) 2050
    Answer: B) 2030
  157. Who made the announcement regarding India’s commitment to debris-free space missions at the 42nd IADC meeting?
    A) Prime Minister Narendra Modi
    B) ISRO Chairman S. Somanath
    C) DRDO Chief G. Satheesh Reddy
    D) Indian Space Minister Jitendra Singh
    Answer: B) ISRO Chairman S. Somanath
  158. Which country recently received the first batch of BrahMos supersonic cruise missiles from India?
    A) Vietnam
    B) Philippines
    C) Indonesia
    D) Malaysia
    Answer: B) Philippines
  159. What is significant about the delivery of BrahMos missiles to the Philippines?
    A) It marks the first export of BrahMos missiles to any country.
    B) It signifies India’s defense cooperation with Malaysia.
    C) It represents the first-ever defense deal between India and Vietnam.
    D) It indicates India’s military alliance with Indonesia.
    Answer: A) It marks the first export of BrahMos missiles to any country.
  160. How much was the deal worth between India and the Philippines for the BrahMos missiles?
    A) $200 million
    B) $300 million
    C) $375 million
    D) $500 million
    Answer: C) $375 million
  161. When was the deal for the BrahMos missiles between India and the Philippines signed?
    A) January 2020
    B) January 2021
    C) January 2022
    D) January 2023
    Answer: C) January 2022
  162. What kind of missiles are BrahMos supersonic cruise missiles?
    A) Ballistic missiles
    B) Anti-aircraft missiles
    C) Cruise missiles
    D) Intercontinental ballistic missiles
    Answer: C) Cruise missiles
  163. Where was the Indigenous Technology Cruise Missile (ITCM) test-fired by DRDO?
    A) Bengaluru
    B) Hyderabad
    C) Chandipur, Odisha
    D) New Delhi
    Answer: C) Chandipur, Odisha
  164. What key features of the Indigenous Technology Cruise Missile (ITCM) were demonstrated during the test?
    A) High-altitude flight and long-range accuracy
    B) Precise navigation, low-altitude sea-skimming flight, and reliability of propulsion
    C) Surface-to-air capability and anti-ship targeting
    D) Submarine-launched deployment and stealth capability
    Answer: B) Precise navigation, low-altitude sea-skimming flight, and reliability of propulsion
  165. Who conducted the successful test flight of the Indigenous Technology Cruise Missile (ITCM)?
    A) Indian Navy
    B) Indian Army
    C) Indian Air Force
    D) Defence Research and Development Organisation (DRDO)
    Answer: D) Defence Research and Development Organisation (DRDO)
  166. Which location in Odisha served as the testing site for the Indigenous Technology Cruise Missile (ITCM)?
    A) Bhubaneswar
    B) Cuttack
    C) Chandipur
    D) Puri
    Answer: C) Chandipur
  167. Which entity closely monitored the performance of the Indigenous Technology Cruise Missile (ITCM) during the test?
    A) Indian Navy
    B) Various sensors
    C) Indian Space Research Organisation (ISRO)
    D) Foreign military observers
    Answer: B) Various sensors
  168. Who shared plans for the next moon mission, Chandrayaan-4, in 2040?
    A) Narendra Modi
    B) S Somanath
    C) Rakesh Sharma
    D) K Sivan
    Answer: B) S Somanath
    Which organization is spearheading India’s lunar exploration efforts?
    A) NASA
    B) ISRO
    C) ESA
    D) CNSA
    Answer: B) ISRO
  169. What is the primary objective of the upcoming phase of the Chandrayaan project?
    A) Exploring Mars
    B) Advancing lunar exploration efforts
    C) Studying asteroids
    D) Investigating exoplanets
    Answer: B) Advancing lunar exploration efforts
  170. What milestone does Chandrayaan-4 aim to achieve for India by 2040?
    A) Establishing a permanent lunar base
    B) Landing an astronaut on the moon
    C) Mapping the entire lunar surface
    D) Extracting water from lunar soil
    Answer: B) Landing an astronaut on the moon
  171. When is the expected timeline for the Chandrayaan-4 mission?
    A) 2030
    B) 2040
    C) 2050
    D) 2060
    Answer: B) 2040
  172. When did Arabica coffee reportedly develop according to genome sequencing?
    A) 6,000 years ago
    B) 60,000 years ago
    C) 600,000 years ago
    D) 6 million years ago
    Answer: C) 600,000 years ago
  173. Where did Arabica coffee develop, according to the report?
    A) Colombia
    B) Ethiopia
    C) Brazil
    D) Indonesia
    Answer: B) Ethiopia
  174. What were the two coffee species involved in the natural hybridization process?
    A) Coffea robusta and Coffea arabica
    B) Coffea arabica and Coffea liberica
    C) Coffea canephora and Coffea eugenioides
    D) Coffea arabica and Coffea excelsa
    Answer: C) Coffea canephora and Coffea eugenioides
  175. What is the aim of India’s ₹10,000 crore Artificial Intelligence Mission?
    A) Boosting the coffee industry
    B) Enhancing the country’s AI ecosystem
    C) Promoting tourism
    D) Supporting traditional farming methods
    Answer: B) Enhancing the country’s AI ecosystem
  176. Which company is India considering striking a deal with for sourcing GPUs?
    A) Intel
    B) AMD
    C) Nvidia
    D) Qualcomm
    Answer: C) Nvidia
  177. Who announced the launch of the new artificial intelligence (AI) assistant, Meta AI?
    A) Google
    B) Apple
    C) Meta Platforms
    D) Microsoft
    Answer: C) Meta Platforms
  178. What is the name of the latest AI model powering Meta AI?
    A) Tiger 5
    B) Llama 3
    C) Elephant 2
    D) Rhino 4
    Answer: B) Llama 3
  179. What is the purpose of Meta AI?
    A) Enhancing virtual reality experiences
    B) Providing users with highly intelligent AI assistance
    C) Developing self-driving cars
    D) Improving mobile gaming performance
    Answer: B) Providing users with highly intelligent AI assistance
  180. Which social media and messaging platforms will integrate Meta AI?
    A) Twitter and LinkedIn
    B) Facebook, Instagram, and WhatsApp
    C) Snapchat and TikTok
    D) Pinterest and Reddit
    Answer: B) Facebook, Instagram, and WhatsApp
  181. What is the pricing strategy for Meta AI?
    A) Premium subscription-based model
    B) Pay-per-use model
    C) Freemium model
    D) Free-to-use model
    Answer: D) Free-to-use model
  182. When was the ‘Project Tiger’ launched by the Government of India?
    a) 1960
    b) 1973
    c) 1985
    d) 1999
    Answer: b) 1973
  183. Which location was chosen as one of the prime locations for Tiger Conservation under ‘Project Tiger’?
    a) Jim Corbett National Park
    b) Gir Forest National Park
    c) Simlipal
    d) Bandhavgarh National Park
    Answer: c) Simlipal
  184. Which zoo has the largest collection of white tigers in India?
    a) Mysore Zoo
    b) Nandankanan Zoological Park
    c) Delhi Zoo
    d) Sanjay Gandhi Biological Park
    Answer: b) Nandankanan Zoological Park
  185. In which year did the Odisha Wildlife Organisation come into existence?
    a) 1968
    b) 1974
    c) 1982
    d) 1990
    Answer: b) 1974
  186. When was the Crocodile Conservation Programme launched in Odisha?
    a) 1960
    b) 1974-75
    c) 1985
    d) 1999
    Answer: b) 1974-75
  187. Which species of crocodile was designated as endangered in 1975?
    a) Nile crocodile
    b) Saltwater crocodile
    c) American crocodile
    d) Indian gharial
    Answer: b) Saltwater crocodile
  188. When was the Sea Turtle Conservation initiative started in Odisha?
    a) 1965
    b) 1976
    c) 1980
    d) 1992
    Answer: b) 1976
  189. What percentage of the total world population of Olive Ridley sea turtles nests along the Odisha coast?
    a) 25%
    b) 50%
    c) 75%
    d) 90%
    Answer: b) 50%
  190. Which beach in Odisha is known as a nesting site for Olive Ridley sea turtles?
    a) Puri Beach
    b) Chandrabhaga Beach
    c) Gahirmatha Beach
    d) Konark Beach
    Answer: c) Gahirmatha Beach
  191. When was ‘Project Elephant’ established in India?
    a) 1973
    b) 1985
    c) 1991
    d) 2000
    Answer: c) 1991
  192. In which city is Nandankanan Zoological Park located?
    A) Kolkata
    B) Bhubaneswar
    C) Chennai
    D) Mumbai
    Answer: B) Bhubaneswar
  193. What is the meaning of “Nandankanan”?
    A) Forest of Chandaka
    B) Garden of Pleasure
    C) Sanctuary of Wildlife
    D) Lake of Migratory Birds
    Answer: B) Garden of Pleasure
  194. When did Nandankanan Zoological Park come into existence?
    A) 1950
    B) 1960
    C) 1970
    D) 1980
    Answer: B) 1960
  195. Which organization did Nandankanan Zoological Park become a member of in 2009?
    A) UNESCO
    B) IUCN
    C) WAZA
    D) WWF
    Answer: C) WAZA (World Association of Zoos and Aquariums)
  196. What is the area covered by Nandankanan Zoological Park?
    A) 40.06 sq km
    B) 4.006 sq km
    C) 400 hectares
    D) 66 hectares
    Answer: B) 4.006 sq km
  197. Which animal was successfully bred in captivity for the first time in India at Nandankanan Zoological Park in 1980?
    A) Indian crocodile
    B) Asiatic lion
    C) Gharial
    D) White tiger
    Answer: C) Gharial
  198. What is the name of the saltwater lake within Nandankanan Zoological Park?
    A) Chandaka Lake
    B) Kania Lake
    C) Nandan Lake
    D) Chandaka Kania Lake
    Answer: B) Kania Lake
  199. Which species served as the first breeding subject at Nandankanan Zoological Park for black panthers?
    A) White tigers
    B) Asiatic lions
    C) Nilgiri langurs
    D) Indian crocodiles
    Answer: A) White tigers
  200. How many species of mammals are found in Nandankanan Zoological Park?
    A) 67
    B) 18
    C) 81
    D) 34
    Answer: A) 67
  201. What attraction is found across the lake in Nandankanan Zoological Park?
    A) Botanical garden
    B) Reptile park
    C) Orchid house
    D) Aquaria
    Answer: A) Botanical garden
  202. Where is Sambalpur Elephant Reserve situated?
    a) Angul
    b) Cuttack
    c) Nayagarh
    d) Sambalpur
    Answer: d) Sambalpur
  203. When did Sambalpur Elephant Reserve come into existence?
    a) 2000
    b) 2002
    c) 2004
    d) 2006
    Answer: b) 2002
  204. What is the total area covered by Sambalpur Elephant Reserve?
    a) 927 sq km
    b) 1,038 sq km
    c) 427 sq km
    d) 537 sq km
    Answer: c) 427 sq km
  205. Which district is not included in the borders of Sambalpur Elephant Reserve?
    a) Angul
    b) Nayagarh
    c) Sundargarh
    d) Dhenkanal
    Answer: c) Sundargarh
  206. Which wildlife sanctuary is a part of Sambalpur Elephant Reserve?
    a) Athagarh Wildlife Sanctuary
    b) Badrama Wildlife Sanctuary
    c) Satkosi Wildlife Sanctuary
    d) Bhitarkanika Wildlife Sanctuary
    Answer: b) Badrama Wildlife Sanctuary
  207. Which forest divisions are included in Sambalpur Elephant Reserve?
    a) Angul, Bamra, Cuttack
    b) Bamra, Bonai, Jharsuguda
    c) Athagarh, Nayagarh, Rairakhol
    d) Satkosi, Mahanadi, Athamalik
    Answer: b) Bamra, Bonai, Jharsuguda
  208. In which year did the reserve come into existence?
    a) 2000
    b) 2001
    c) 2002
    d) 2003
    Answer: c) 2002
  209. How many districts border the Sambalpur Elephant Reserve?
    a) Three
    b) Four
    c) Five
    d) Six
    Answer: b) Four
  210. Which of the following is not a district sharing borders with Sambalpur Elephant Reserve?
    a) Cuttack
    b) Nayagarh
    c) Sundargarh
    d) Dhenkanal
    Answer: c) Sundargarh
  211. Which forest division is entirely included in Sambalpur Elephant Reserve?
    a) Athagarh
    b) Satkosi
    c) Mahanadi
    d) Bamra
    Answer: b) Satkosi
  212. Which state recently received a GI tag for “Koraput Kala Jeera Rice”?
    a) Odisha
    b) West Bengal
    c) Karnataka
    d) Assam
    Answer: a) Odisha
  213. What category does “Orissa Ikat” belong to, which recently received a GI tag?
    a) Handicraft
    b) Handloom
    c) Cuisine
    d) Agricultural product
    Answer: a) Handicraft
  214. “Similipal Kai chutney” is a traditional delicacy from which district of Odisha?
    a) Kalahandi
    b) Rayagada
    c) Mayurbhanj
    d) Nayagarh
    Answer: c) Mayurbhanj
  215. Which tribal community is associated with the traditional Lanjia Saura paintings?
    a) Dongria Kondh
    b) Lanjia Saura
    c) Nayagarh Kantaimundi
    d) Gajapati
    Answer: b) Lanjia Saura
  216. The “Kantaimundi Brinjal” is primarily grown in which district of Odisha?
    a) Rayagada
    b) Koraput
    c) Nayagarh
    d) Mayurbhanj
    Answer: c) Nayagarh
  217. What is the primary ingredient used to make “Odisha Khajuri Guda”?
    a) Date palm
    b) Coconut
    c) Jaggery
    d) Milk
    Answer: a) Date palm
  218. “Dhenkanal Magji” is a dessert originating from which district of Odisha?
    a) Gajapati
    b) Dhenkanal
    c) Rayagada
    d) Mayurbhanj
    Answer: b) Dhenkanal
  219. Which tribal community is known for crafting the “Kapdaganda shawl” in Odisha?
    a) Dongria Kondh
    b) Lanjia Saura
    c) Nayagarh Kantaimundi
    d) Similipal Kai
    Answer: a) Dongria Kondh
  220. What is the significance of the yellow thread used in the “Kapdaganda shawl”?
    a) Represents mountains
    b) Symbolizes blood
    c) Signifies calm and pleasure
    d) Represents the Sun
    Answer: c) Signifies calm and pleasure
  221. Which region of Odisha is known for its “Off-white coarse cotton” used in the “Kapdaganda shawl”?
    a) Nayagarh
    b) Kalahandi
    c) Rayagada
    d) Dhenkanal
    Answer: c) Rayagada
  222. What is the primary characteristic of “Koraput Kala Jeera Rice”?
    a) Small size
    b) Red color
    c) Aroma similar to cumin seeds
    d) High starch content
    Answer: c) Aroma similar to cumin seeds
  223. Which element is depicted most frequently in the “Lanjia Saura paintings”?
    a) Mountains
    b) Animals
    c) Trees
    d) Rivers
    Answer: c) Trees
  224. How do tribes in Mayurbhanj prepare “Similipal Kai chutney”?
    a) Boiling
    b) Grinding red ants
    c) Roasting
    d) Sun-drying
    Answer: b) Grinding red ants
  225. What is the characteristic feature of “Nayagarh Kantaimundi Brinjal”?
    a) Round shape
    b) Thorny stems
    c) Purple color
    d) Sweet taste
    Answer: b) Thorny stems
  226. Which dessert has its origin traced back to the British era in Odisha?
    a) Odisha Khajuri Guda
    b) Dhenkanal Magji
    c) Similipal Kai chutney
    d) Nayagarh Kantaimundi Brinjal
    Answer: b) Dhenkanal Magji
  227. Rupa Tarakasi, which recently received a GI tag, is associated with which city in India?
    a) Jaipur
    b) Cuttack
    c) Varanasi
    d) Kolkata
    Answer: b) Cuttack
  228. What craft does the GI tag “Rupa Tarakasi” signify?
    a) Pottery
    b) Weaving
    c) Silver Filigree
    d) Embroidery
    Answer: c) Silver Filigree
  229. Which of the following is true regarding the recently GI-tagged “Rupa Tarakasi”?
    a) It originated in Jaipur
    b) It is known for its wooden carvings
    c) It signifies the craftsmanship of Cuttack
    d) It is primarily associated with Kolkata
    Answer: c) It signifies the craftsmanship of Cuttack
  230. Which of the following is not a characteristic of Nayagarh Kantaimundi Brinjal?
    a) Thorny stems
    b) Spherical fruits
    c) High susceptibility to insects
    d) Grown in hilly locations
    Answer: c) High susceptibility to insects
  231. What is the primary nutritional benefit associated with consuming Similipal Kai chutney?
    a) Boosts haemoglobin levels
    b) Improves metabolism
    c) Rich in protein
    d) Provides essential vitamins and minerals
    Answer: c) Rich in protein
  232. Which tribe is known for making Kapdaganda shawls?
    a) Lanjia Saura
    b) Nayagarh Kantaimundi
    c) Dongria Kondh
    d) Koraput Kala Jeera
    Answer: c) Dongria Kondh
  233. What does the red color in the Kapdaganda shawl symbolize?
    a) Mountains and hills
    b) Calm and pleasure
    c) Love and affection for nature
    d) Blood
    Answer: d) Blood
  234. What color background is typically used in Lanjia Saura paintings?
    a) White
    b) Red
    c) Green
    d) Yellow
    Answer: a) White
  235. What is another name for Koraput Kala Jeera rice?
    a) Black rice
    b) Prince of Rice
    c) Cumin rice
    d) Kala Jeera rice
    Answer: b) Prince of Rice
  236. What is the primary ingredient in Similipal Kai chutney?
    a) Brinjal
    b) Date palm jaggery
    c) Red weaver ants
    d) Buffalo milk cheese
    Answer: c) Red weaver ants
  237. What is the origin of Odisha Khajuri Guda?
    a) Gajapati area
    b) Nayagarh district
    c) Rayagada district
    d) Mayurbhanj district
    Answer: a) Gajapati area
  238. What is the significance of yellow in the Kapdaganda shawl?
    a) Represents calm and pleasure
    b) Symbolizes mountains and hills
    c) Emblem of blood
    d) Represents love and affection for nature
    Answer: a) Represents calm and pleasure
  239. Dhenkanal Magji, a dessert, is made from which type of milk?
    a) Cow milk
    b) Goat milk
    c) Buffalo milk
    d) Camel milk
    Answer: c) Buffalo milk
  240. “Kendrapara Rasabali,” recently granted a GI tag, originates from which Indian state?
    a) Odisha
    b) West Bengal
    c) Tamil Nadu
    d) Maharashtra
    Answer: a) Odisha
  241. In which category does “Kendrapara Rasabali” fall under, as indicated by its recent GI designation?
    a) Handicraft
    b) Cuisine
    c) Textile
    d) Agricultural product
    Answer: b) Cuisine
  242. The GI tag for “Rasabali” signifies the unique heritage and culinary tradition of which specific region?
    a) Cuttack
    b) Bhubaneswar
    c) Kendrapara
    d) Puri
    Answer: c) Kendrapara
  243. The NUA-O Scholarship 2024 initiative, aimed at providing annual scholarships to undergraduate and postgraduate students, is introduced by the government of which Indian state?
    a) West Bengal
    b) Odisha
    c) Tamil Nadu
    d) Uttar Pradesh
    Answer: b) Odisha
  244. Under the NUA-O Scholarship 2024, what is the annual scholarship amount allocated for male students?
    a) Rs 8,000
    b) Rs 9,000
    c) Rs 10,000
    d) Rs 11,000
    Answer: b) Rs 9,000
  245. Which category of students are eligible to benefit from the NUA-O Scholarship 2024?
    a) Primary school students
    b) High school students
    c) Undergraduate and postgraduate students
    d) PhD scholars
    Answer: c) Undergraduate and postgraduate students
  246. What is the purpose of introducing the NUA-O Scholarship 2024 by the Odisha government?
    a) To promote sports among students
    b) To encourage entrepreneurship
    c) To provide financial assistance to students for their education
    d) To support cultural activities
    Answer: c) To provide financial assistance to students for their education
  247. How much scholarship amount will female students receive annually under the NUA-O Scholarship 2024?
    a) Rs 8,000
    b) Rs 9,000
    c) Rs 10,000
    d) Rs 11,000
    Answer: c) Rs 10,000
  248. When did the Chinese traveler Hiuen Tsang visit Odisha?
    [A] 620-621 AD
    [B] 683-684 AD
    [C] 638-639 AD
    [D] 650-651 AD
    Answer: C [638-639 AD]
  249. During his visit to Odisha, which port did Hiuen Tsang describe as a bustling center?
    [A] Puri
    [B] Chelitalo
    [C] Cuttack
    [D] Paradip
    Answer: B [Chelitalo]
  250. What is the name of the new scheme launched by the Odisha government in 2024 to provide interest-free loans to budding entrepreneurs?
    a) LABHA Yojana
    b) Swayam Scheme
    c) Balaram Yojana
    d) Banishree Yojana
    Answer: b) Swayam Scheme
  251. Under the Swayam Scheme 2024, how much interest-free loan is provided to budding entrepreneurs?
    a) Rs 50,000
    b) Rs 75,000
    c) Rs 1,00,000
    d) Rs 1,50,000
    Answer: c) Rs 1,00,000
  252. Which sector does the Swayam Scheme 2024 primarily aim to support?
    a) Agriculture
    b) Education
    c) Healthcare
    d) Social Security
    Answer: b) Education
  253. When was the Banishree Yojana launched by the Odisha government?
    a) 2007
    b) 2008
    c) 2010
    d) 2012
    Answer: b) 2008
  254. Which of the following schemes focuses on providing social security benefits?
    a) Banishree Yojana
    b) Swayam Scheme
    c) Madhu Babu Pension Yojana
    d) Kishori Shakti Yojana
    Answer: c) Madhu Babu Pension Yojana
  255. What is the objective of the Balaram Yojana launched by the Odisha government?
    a) Providing interest-free loans to farmers
    b) Facilitating institutional credit facilities to sharecroppers
    c) Promoting entrepreneurship among women
    d) Supporting education for underprivileged children
    Answer: b) Facilitating institutional credit facilities to sharecroppers
  256. Which scheme focuses on empowering women in Odisha?
    a) Banishree Yojana
    b) Madhu Babu Pension Yojana
    c) Vasundhara Scheme
    d) Kishori Shakti Yojana
    Answer: d) Kishori Shakti Yojana
  257. When was the Vasundhara Scheme launched by the Odisha government?
    a) 2007
    b) 2008
    c) 2010
    d) 2012
    Answer: b) 2008
  258. Which scheme focuses on providing education-related benefits in Odisha?
    a) Banishree Yojana
    b) Madhu Babu Pension Yojana
    c) Vasundhara Scheme
    d) Kishori Shakti Yojana
    Answer: a) Banishree Yojana
  259. What does the LABHA Yojana introduced by the Odisha government primarily aim to support?
    a) Healthcare facilities
    b) Minimum support price for agricultural produce
    c) Social security for senior citizens
    d) Minimum support price for minor forest produce
    Answer: d) Minimum support price for minor forest produce
  260. Which court appointed a panel to assess the condition of leprosy patients in Odisha?
    a) Supreme Court of India
    b) High Court of Odisha
    c) High Court of India
    d) District Court of Odisha
    Answer: b) High Court of Odisha
  261. What is the significance of the Gupteswar Forest in Odisha?
    a) It is known for its biodiversity and declared as a Biodiversity Heritage Site
    b) It is a historical site related to ancient rulers
    c) It is a popular tourist destination for adventure sports
    d) It is a major industrial hub in the state
    Answer: a) It is known for its biodiversity and declared as a Biodiversity Heritage Site
  262. What is the objective of the LABHA Yojana introduced by the Odisha government?
    a) To provide interest-free loans to budding entrepreneurs
    b) To provide social security benefits to senior citizens
    c) To provide minimum support price for minor forest produce
    d) To facilitate institutional credit facilities to sharecroppers
    Answer: c) To provide minimum support price for minor forest produce
  263. Which scheme focuses on providing pensions to senior citizens in Odisha?
    a) Banishree Yojana
    b) Madhu Babu Pension Yojana
    c) Vasundhara Scheme
    d) Kishori Shakti Yojana
    Answer: b) Madhu Babu Pension Yojana
  264. What does the acronym “LABHA” stand for in the LABHA Yojana launched by the Odisha government?
    a) Laghu Bana Jatya Drabya Kraya
    b) Laghu Bana Jatya Development Scheme
    c) Laghu Bana Janata Diwas
    d) Laghu Bana Jatya Financial Assistance
    Answer: a) Laghu Bana Jatya Drabya Kraya
  265. What is the full form of “BALARAM” in the scheme launched by the Odisha government?a) Balancing Rural Agriculture through Resources and Management
    b) Bhoomihina Agriculturist Loan and Resources Augmentation Model
    c) Boosting Agricultural Livelihoods and Resources Access Model
    d) Basic Agricultural Land Resources Augmentation ModelAnswer: b) Bhoomihina Agriculturist Loan and Resources Augmentation Model
  266. Which district in Odisha is home to the recently designated Gupteswar forest, declared as the fourth Biodiversity Heritage Site?
    a) Cuttack
    b) Koraput
    c) Puri
    d) Ganjam
    Answer: b) Koraput
    Which forest in Odisha was recently declared as the fourth Biodiversity Heritage Site?a) Mahendragiri Forest
    b) Gandhamardan Forest
    c) Gupteswar Forest
    d) Mandasaru ForestAnswer: c) Gupteswar Forest
  267. Which port recently emerged as the highest cargo handling port in India?
    a) Mumbai Port
    b) Paradip Port
    c) Chennai Port
    d) Deendayal Port, Kandla
    Answer:b) Paradip Port
  268. What was the total cargo handling capacity achieved by the Paradip Port Authority (PPA) in the fiscal year 2023-24?
    a) 135.38 million metric tonnes
    b) 145.38 million metric tonnes
    c) 155.38 million metric tonnes
    d) 165.38 million metric tonnes
    Answer: b) 145.38 million metric tonnes
  269. How many years has it been since the Paradip Port Authority (PPA) surpassed the previous record set by Deendayal Port in terms of total cargo handling?
    a) 20 years
    b) 36 years
    c) 50 years
    d) 56 years
    Answer: d) 56 years
  270. Which port previously held the record for the highest cargo handling in India before being surpassed by the Paradip Port Authority (PPA) in the fiscal year 2023-24?
    a) Mumbai Port
    b) Chennai Port
    c) Kolkata Port
    d) Deendayal Port, Kandla
    Answer: d) Deendayal Port, Kandla
  271. Shantilata Barik, who recently passed away, belonged to which Indian state?
    a) Kerala
    b) Odisha
    c) West Bengal
    d) Karnataka
    Answer: b) Odisha
  272. In which field did Shantilata Barik gain recognition and acclaim?
    a) Literature
    b) Sports
    c) Music
    d) Politics
    Answer: c) Music
  273. Which ancient supercontinent does the Indian subcontinent, including Odisha, originate from?
    A) Pangea
    B) Gondwana
    C) Laurasia
    D) Rodinia
    Answer: B) Gondwana
  274. What are the four main natural divisions of Odisha?
    A) Western Ghats, Central Highlands, Eastern Ghats, Coastal Plains
    B) Northern Plateau, Southern Plateau, Coastal Plains, Eastern Ghats
    C) Northern Plateau, Eastern Ghats, Central Tract, Coastal Plains
    D) Northern Plains, Central Plains, Southern Plains, Coastal Plains
    Answer: C) Northern Plateau, Eastern Ghats, Central Tract, Coastal Plains
  275. What is the approximate elevation of the Eastern Ghats in Odisha?
    A) 2,500 feet (762 meters)
    B) 3,000 feet (914 meters)
    C) 3,600 feet (1,100 meters)
    D) 4,000 feet (1,219 meters)
    Answer: C) 3,600 feet (1,100 meters)
  276. Which river is NOT one of the main rivers in Odisha?
    A) Mahanadi
    B) Subarnarekha
    C) Godavari
    D) Brahmani
    Answer: C) Godavari
  277. Which of the following is NOT a division of the coastal plains in Odisha?
    A) Balasore coastal plain
    B) Mahanadi River delta
    C) Chilka plain
    D) Vamsadhara delta
    Answer: D) Vamsadhara delta
  278. Which lake in Odisha is one of the largest lagoons in India?
    A) Vembanad Lake
    B) Dal Lake
    C) Chilka Lake
    D) Pulicat Lake
    Answer: C) Chilka Lake
  279. What is the highest peak in Odisha?
    A) Mahendra Giri
    B) Malayagiri
    C) Megasini
    D) Satkosia
    Answer: A) Mahendra Giri
  280. Which of the following areas in Odisha is known for its mineral-rich deposits?
    A) Central Tract
    B) Eastern Ghats
    C) Coastal Plains
    D) Northern Plateau
    Answer: D) Northern Plateau
  281. Which river forms the Mahanadi River delta in Odisha?
    A) Subarnarekha
    B) Brahmani
    C) Baitarani
    D) Mahanadi itself
    Answer: D) Mahanadi itself
  282. What type of deposits are found along the seaboard in Odisha?
    A) Glacial deposits
    B) Volcanic deposits
    C) Deltaic alluvial deposits
    D) Karst deposits
    Answer: C) Deltaic alluvial deposits
  283. What climatic region is Odisha located in?
    A) Tropical rainforest
    B) Tropical wet-dry (or tropical savanna)
    C) Mediterranean
    D) Temperate
    Answer: B) Tropical wet-dry (or tropical savanna)
  284. In which month does Cuttack experience the coolest temperatures?
    A) May
    B) June
    C) January
    D) September
    Answer: C) January
  285. What is the typical high temperature in Cuttack during January?
    A) Mid-90s F
    B) Mid-80s F
    C) Mid-70s F
    D) Mid-60s F
    Answer: B) Mid-80s F
  286. During which month does Cuttack usually experience the warmest temperatures?
    A) May
    B) June
    C) July
    D) August
    Answer: A) May
  287. What is the average low temperature in Cuttack during May?
    A) Low 90s F
    B) Low 80s F
    C) Low 70s F
    D) Low 60s F
    Answer: C) Low 70s F
  288. Which geographical feature provides some relief from the summer heat in Odisha?
    A) Plateaus
    B) Plains
    C) Valleys
    D) Hills
    Answer: D) Hills
  289. During which months does the southwest monsoon occur in Odisha?
    A) December through February
    B) March through May
    C) June through September
    D) October through November
    Answer: C) June through September
  290. What is the average annual rainfall in Odisha?
    A) 40 inches (1,000 mm)
    B) 60 inches (1,500 mm)
    C) 80 inches (2,000 mm)
    D) 100 inches (2,500 mm)
    Answer: B) 60 inches (1,500 mm)
  291. Which region of Odisha receives heavier precipitation?
    A) Coastal area south of Chilka Lake
    B) Western plains
    C) Eastern Ghats
    D) Central tract basins
    Answer: C) Eastern Ghats
  292. Which area in Odisha is considered the driest region in the state?
    A) Coastal area south of Chilka Lake
    B) Western plains
    C) Eastern Ghats
    D) Central tract basins
    Answer: A) Coastal area south of Chilka Lake
  293. Which category of forests covers nearly one-third of Odisha’s land area?
    a) Tropical rainforests
    b) Tropical moist deciduous and tropical dry deciduous
    c) Coniferous forests
    d) Temperate forests
    Answer: b) Tropical moist deciduous and tropical dry deciduous
  294. Where is the tropical moist deciduous forest primarily located in Odisha?
    a) Southwest
    b) Southeast
    c) Northeast
    d) Northwest
    Answer: c) Northeast
  295. Which type of forests is found in the southwest of Odisha?
    a) Tropical rainforests
    b) Tropical moist deciduous
    c) Tropical dry deciduous
    d) Coniferous forests
    Answer: c) Tropical dry deciduous
  296. What is a common plant species found in both tropical moist deciduous and tropical dry deciduous forests of Odisha?
    a) Pine
    b) Bamboo
    c) Eucalyptus
    d) Oak
    Answer: b) Bamboo
  297. Which of the following is NOT a type of tropical hardwood found in Odisha’s forests?
    a) Teak
    b) Rosewood
    c) Padauk
    d) Pine
    Answer: d) Pine
  298. What is the general trend of forest cover density from northeast to southwest in Odisha?
    a) Increases
    b) Decreases
    c) Remains constant
    d) Fluctuates
    Answer: b) Decreases
  299. Which of the following mammals is NOT commonly found in Odisha’s forests?
    a) Elephants
    b) Tigers
    c) Lions
    d) Gaurs (wild cattle)
    Answer: c) Lions
  300. What bird species is characteristic of Odisha’s forests?
    a) Eagles
    b) Peacocks
    c) Owls
    d) Falcons
    Answer: b) Peacocks
  301. Which coastal feature in Odisha serves as a breeding ground for many fish and waterfowl?
    a) Mahanadi River
    b) Subarnarekha River
    c) Chilka Lake
    d) Bhitarkanika Mangroves
    Answer: c) Chilka Lake
  302. What governmental bodies establish parks and sanctuaries to protect wildlife in Odisha?
    a) Local municipalities
    b) International organizations
    c) State and national governments
    d) Non-governmental organizations (NGOs)
    Answer: c) State and national governments
  303. Which dynasty of Odisha saw the reign of six widowed queens and one unmarried princess?
    [A] Bhauma Kara
    [B] Shilod Bhava
    [C] Mudgals
    [D] Somavamsi
    Answer: A [Bhauma Kara]
  304. In 1849, which part of Odisha came under direct British control through the application of the Doctrine of Lapse?
    [A] Satara
    [B] Cuttack
    [C] Sambalpur
    [D] Koraput
    Answer: C [Sambalpur]
  305. During which dynasty of Odisha did queens enjoy successful reigns?
    [A] Somavamsi
    [B] Shilod Bhava
    [C] Bhauma Kara
    [D] Gajapati
    Answer: C [Bhauma Kara]
  306. Which doctrine, introduced by Lord Dalhousie, led to the direct British control of Sambalpur in 1849?
    [A] Doctrine of Divinity
    [B] Doctrine of Sovereignty
    [C] Doctrine of Lapse
    [D] Doctrine of Exclusion
    Answer: C [Doctrine of Lapse]
  307. Under which dynasty did Odisha witness the rule of queens such as Tribhuvana Mahadevi?
    [A] Gajapati
    [B] Shilod Bhava
    [C] Bhauma Kara
    [D] Somavamsi
    Answer: C [Bhauma Kara]
  308. Who served as the first Governor of Odisha?
    [A] Sir John Austen Hubback
    [B] Sir Chandulal Madhavlal Trivedi
    [C] Saiyid Nurul Hasan
    [D] Bishambhar Nath Pande
    Answer: A [Sir John Austen Hubback]
  309. During which period did Sir John Austen Hubback serve as the Governor of Odisha?
    [A] April 1936 to August 1938
    [B] December 1938 to March 1941
    [C] Both A and B
    [D] None of the above
    Answer: C [Both A and B]
  310. How long is the term of the Governor of Odisha?
    [A] 3 years
    [B] 4 years
    [C] 5 years
    [D] 6 years
    Answer: C [5 years]
  311. Who was the first woman Chief Minister of Odisha?
    [A] Nandini Satpathy
    [B] Sushama Swaraj
    [C] Mayawati
    [D] Mamata Banerjee
    Answer: A [Nandini Satpathy]
  312. Who was the first Odia woman to serve as a Governor?
    [A] Ramadevi
    [B] Vimala Sharma
    [C] Sarojini Naidu
    [D] Sucheta Kriplani
    Answer: A [Ramadevi]
  313. In which state did Ramadevi serve as the Governor?
    [A] Karnataka
    [B] Odisha
    [C] West Bengal
    [D] Tamil Nadu
    Answer: A [Karnataka]
  314. Apart from being the first Odia lady Governor, what other notable position did Ramadevi hold?
    [A] Chief Minister
    [B] Chief Justice
    [C] Chief Election Commissioner
    [D] President
    Answer: C [Chief Election Commissioner]
  315. Who was the first Governor of Odisha after India’s independence?
    [A] Kailash Nath Katju
    [B] Bishweshwar Nath Katju
    [C] Ganeshi Lal Mathur
    [D] Vimala Sharma
    Answer: A [Kailash Nath Katju]
  316. Who is the current Governor of Odisha (as of 2020)?
    [A] Kailash Nath Katju
    [B] Bishweshwar Nath Katju
    [C] Ganeshi Lal Mathur
    [D] Raghubar Das
    Answer: [D] Raghubar Das
  317. Who was the first woman IAS officer in Odisha?
    [A] Sujata R Karthikeyan
    [B] Vimala Sharma
    [C] Kiran Bedi
    [D] Anna Rajam Malhotra
    Answer: A [Sujata R Karthikeyan]
  318. In which district of Odisha does Sujata R Karthikeyan hail from?
    [A] Cuttack
    [B] Kendrapara
    [C] Puri
    [D] Bhubaneswar
    Answer: B [Kendrapara]
  319. Who was the first Chief Minister of India?
    [A] Jawaharlal Nehru
    [B] Sardar Vallabhbhai Patel
    [C] Pandit Govind Ballabh Pant
    [D] Lal Bahadur Shastri
    Answer: C [Pandit Govind Ballabh Pant]
  320. In which province did Pandit Govind Ballabh Pant serve as the first Chief Minister?
    [A] Uttar Pradesh
    [B] Bihar
    [C] Madhya Pradesh
    [D] Rajasthan
    Answer: A [Uttar Pradesh]
  321. During which years did Pandit Govind Ballabh Pant serve as the Chief Minister of Uttar Pradesh?
    [A] 1937 and 1950
    [B] 1947 and 1960
    [C] 1950 and 1970
    [D] 1960 and 1980
    Answer: A [1937 and 1950]
  322. Who is the 26th Governor of Odisha?
    [A] Raghubar Das
    [B] Ganeshi Lal Mathur
    [C] Kailash Nath Katju
    [D] Bishweshwar Nath Katju
    Answer: A [Raghubar Das]
  323. Who succeeded Ganeshi Lal Mathur as the Governor of Odisha?
    [A] Raghubar Das
    [B] Vajubhai Vala
    [C] Satya Pal Malik
    [D] Prof. Ganeshi Lal Mathur
    Answer: A [Raghubar Das]
  324. Where was Droupadi Murmu born?
    [A] Uparbeda village, Mayurbhanj, Odisha
    [B] Bhubaneswar, Odisha
    [C] Rairangpur, Odisha
    [D] Uparbeda village, Rairangpur, Odisha
    Answer: A [Uparbeda village, Mayurbhanj, Odisha]
  325. What was the original name of Droupadi Murmu before she was named Droupadi by a teacher?
    [A] Biranchi Narayan Tudu
    [B] Puti Tudu
    [C] Sarpanch
    [D] Mayurbhanj
    Answer: B [Puti Tudu]
  326. Who was the first tribal woman to serve as the President of India?
    [A] Pratibha Patil
    [B] Droupadi Murmu
    [C] Sarojini Naidu
    [D] Indira Gandhi
    Answer: B [Droupadi Murmu]
  327. Which state did Droupadi Murmu belong to?
    [A] Odisha
    [B] Bihar
    [C] Jharkhand
    [D] West Bengal
    Answer: A [Odisha]
  328. In which area did Droupadi Murmu’s father and grandfather serve as traditional heads of the village council?
    [A] Bhubaneswar
    [B] Mayurbhanj
    [C] Rairangpur
    [D] Uparbeda village
    Answer: D [Uparbeda village]
  329. On which date is Odisha Day, or Utkala Dibasa, celebrated?
    [A] April 1
    [B] May 1
    [C] June 1
    [D] July 1
    Answer: A [April 1]
  330. Why is April 1 celebrated as Odisha Day?
    [A] It marks the formation of Odisha as a separate state.
    [B] It commemorates the birth of a famous Odia poet.
    [C] It signifies the beginning of the Odia New Year.
    [D] It celebrates the victory of Odisha in a historical battle.
    Answer: A [It marks the formation of Odisha as a separate state.]
  331. In which year was Odisha carved out as a separate state from the erstwhile Bihar and Orissa Province?
    [A] 1915
    [B] 1920
    [C] 1936
    [D] 1947
    Answer: C [1936]
  332. Which districts were added to Odisha during its formation in 1936?
    [A] Koraput and Ganjam
    [B] Cuttack and Puri
    [C] Balasore and Mayurbhanj
    [D] Bhubaneswar and Rourkela
    Answer: A [Koraput and Ganjam]
  333. In which year did the Portuguese establish settlements at Pipili (Balasore district)?
    [A] 1625 AD
    [B] 1633 AD
    [C] 1765 AD
    [D] 1803 AD
    Correct Answer: A [1625 AD]
  334. When did the First British settlement occur at Hariharpur?
    [A] 1625 AD
    [B] 1633 AD
    [C] 1765 AD
    [D] 1803 AD
    Correct Answer: B [1633 AD]
  335. Who acquired the Diwani rights of Odisha in 1765 AD?
    [A] Robert Clive
    [B] Major Broughton
    [C] Bakshi Jagabandhu
    [D] Surendra Sai
    Correct Answer: A [Robert Clive]
  336. In which year did the English occupy Barabati Fort near Cuttack?
    [A] 1765 AD
    [B] 1803 AD
    [C] 1804 AD
    [D] 1833 AD
    Correct Answer: B [1803 AD]
  337. The Khorda Rebellion was led by whom in 1804 AD?
    [A] Robert Clive
    [B] Bakshi Jagabandhu
    [C] Jayee Krushna Rajguru
    [D] Surendra Sai
    Correct Answer: C [Jayee Krushna Rajguru]
  338. Who acquired Sambalpur in 1804 AD?
    [A] Major Broughton
    [B] Robert Clive
    [C] Bakshi Jagabandhu
    [D] Chandan Hajuri
    Correct Answer: A [Major Broughton]
  339. In which year did the Paika Rebellion occur?
    [A] 1817 AD
    [B] 1825 AD
    [C] 1833 AD
    [D] 1857 AD
    Correct Answer: A [1817 AD]
  340. When did Bakshi Jagabandhu surrender?
    [A] 1817 AD
    [B] 1825 AD
    [C] 1833 AD
    [D] 1857 AD
    Correct Answer: B [1825 AD]
  341. The Revolt in Sambalpur by Surendra Sai occurred in which year?
    [A] 1817 AD
    [B] 1825 AD
    [C] 1833 AD
    [D] 1857 AD
    Correct Answer: C [1833 AD]
  342. In which year did the Ghumsar Rebellion start?
    [A] 1817 AD
    [B] 1825 AD
    [C] 1833 AD
    [D] 1857 AD
    Correct Answer: C [1833 AD]
  343. Surendra Sai, Chandan Hajuri, and Ramakrishna Samantasinghar joined the Sepoy Mutiny in which year?
    [A] 1835 AD
    [B] 1865-66 AD
    [C] 1867 AD
    [D] 1868 AD
    Correct Answer: A [1835 AD]
  344. When did the Na’anka Famine occur?
    [A] 1865-66 AD
    [B] 1867 AD
    [C] 1868 AD
    [D] 1869 AD
    Correct Answer: A [1865-66 AD]
  345. The opening of College in Cuttack took place in which year?
    [A] 1865-66 AD
    [B] 1867 AD
    [C] 1868 AD
    [D] 1869 AD
    Correct Answer: B [1867 AD]
  346. The Bhuyan Uprising happened in which year?
    [A] 1865-66 AD
    [B] 1867 AD
    [C] 1868 AD
    [D] 1869 AD
    Correct Answer: C [1868 AD]
  347. In which year was the Utkal Brahmo Samaj formed?
    [A] 1882 AD
    [B] 1896 AD
    [C] 1868 AD
    [D] 1869 AD
    Correct Answer: A [1882 AD]
  348. In which year was the Utkal Union Conference formed?
    [A] 1903 AD
    [B] 1905 AD
    [C] 1909 AD
    [D] 1921 AD
    Correct Answer: A [1903 AD]
  349. What event occurred in Odisha in 1905 AD?
    [A] Formation of Utkal Union Samillani
    [B] Formation of Satyabadi Vana Vidyalaya
    [C] Peasant Revolt at Kanika
    [D] Odisha was granted status of a province
    Correct Answer: B [Formation of Utkal Union Samillani]
  350. When was the Satyabadi Vana Vidyalaya established at Puri?
    [A] 1905 AD
    [B] 1909 AD
    [C] 1921 AD
    [D] 1922 AD
    Correct Answer: B [1909 AD]
  351. Mahatma Gandhi’s first visit to Odisha occurred in which year?
    [A] 1905 AD
    [B] 1909 AD
    [C] 1921 AD
    [D] 1922 AD
    Correct Answer: C [1921 AD]
  352. What event took place in 1922 AD in Odisha?
    [A] Peasant Revolt at Kanika
    [B] Foot March towards Inchudi at Balasore
    [C] Satyagrahis manufactured salt at Inchudi
    [D] Formation of Odisha State’s People’s Conference
    Correct Answer: A [Peasant Revolt at Kanika]
  353. On which date did the Foot March towards Inchudi at Balasore begin in 1930 AD?
    [A] 6th April
    [B] 14th April
    [C] 1st April
    [D] 1st December
    Correct Answer: A [6th April]
  354. When did the Satyagrahis manufacture salt at Inchudi?
    [A] 6th April 1930 AD
    [B] 14th April 1930 AD
    [C] 1931 AD
    [D] 1st April 1936 AD
    Correct Answer: B [14th April 1930 AD]
  355. In which year was the Odisha State’s People’s Conference formed?
    [A] 1921 AD
    [B] 1922 AD
    [C] 1930 AD
    [D] 1931 AD
    Correct Answer: D [1931 AD]
  356. When was Odisha granted the status of a province?
    [A] 1903 AD
    [B] 1921 AD
    [C] 1931 AD
    [D] 1st April 1936 AD
    Correct Answer: D [1st April 1936 AD]
  357. Prajamandal was formed in which year?
    [A] 1931 AD
    [B] 1936 AD
    [C] 1938 AD
    [D] 1940 AD
    Correct Answer: C [1938 AD]
  358. What movement was launched in Odisha on 1st December 1940 AD?
    [A] Satyagraha Movement
    [B] Quit India Movement
    [C] Individual Civil Disobedience Movement
    [D] Non-Cooperation Movement
    Correct Answer: C [Individual Civil Disobedience Movement]
  359. When was the Coalition Ministry formed with the Maharaja of Paralakhemundi?
    [A] November 1941 AD
    [B] 28th September 1942 AD
    [C] 29th March 1943 AD
    [D] December 1947 AD
    Correct Answer: A [November 1941 AD]
  360. The Massacre of Eram in Basudevpur occurred on which date?
    [A] 28th September 1942 AD
    [B] 29th March 1943 AD
    [C] December 1947 AD
    [D] 1st January 1948 AD
    Correct Answer: A [28th September 1942 AD]
  361. On 29th March 1943 AD, who was executed in Behrampur Jail?
    [A] Mahatma Gandhi
    [B] Martyr Laxman Naik
    [C] Surendra Sai
    [D] Sardar Patel
    Correct Answer: B [Martyr Laxman Naik]
  362. When did Sardar Patel come to Odisha for its merger?
    [A] December 1947 AD
    [B] 1st January 1948 AD
    [C] 1949 AD
    [D] 1948 AD
    Correct Answer: A [December 1947 AD]
  363. After the invasion of Samudragupta, who rose to power in the coastal belt of Odisha?
    [A] Matharas
    [B] Gangas
    [C] Nalas
    [D] Meghas
    Correct Answer: A [Matharas]
    Explanation: The Matharas gained control immediately following Samudragupta’s campaign. Shaktivarmana, a ruler of the Mathara dynasty, was titled “Lord of Kalinga.”
  364. Which dynasty came into prominence in the coastal region of Odisha after the conquest by Samudragupta?
    [A] Cholas
    [B] Pallavas
    [C] Matharas
    [D] Chalukyas
    Correct Answer: C [Matharas]
    Explanation: Following Samudragupta’s conquest, the Mathara dynasty rose to power in the coastal region of Odisha, with rulers like Shaktivarmana assuming the title “Lord of Kalinga.”
  365. Who established dominance in the coastal belt of Odisha subsequent to Samudragupta’s invasion?
    [A] Gangas
    [B] Satavahanas
    [C] Matharas
    [D] Cheras
    Correct Answer: C [Matharas]
    Explanation: The Mathara dynasty ascended to prominence in the coastal belt of Odisha after Samudragupta’s invasion, with rulers such as Shaktivarmana exerting control and adopting the title “Lord of Kalinga.”
  366. Which deity did the Sailodvabhavas primarily follow?
    [A] Shaivism
    [B] Vaishnavism
    [C] Buddhism
    [D] Jainism
    Correct Answer: A [Shaivism]
    Explanation: The Sailodvabhavas were followers of Shaivism, as evidenced by their construction of the Parashurameshvara Shiva temple in Bhubaneswar.
  367. What was the main deity worshipped by the Sailodvabhavas?
    [A] Vishnu
    [B] Shiva
    [C] Brahma
    [D] Durga
    Correct Answer: B [Shiva]
    Explanation: Sailodvabhavas primarily worshipped Shiva, as indicated by their adherence to Shaivism and the construction of Shiva temples like the Parashurameshvara temple.
  368. During the Ganga period, which religion was prominent among the rulers of Odisha?
    [A] Hinduism
    [B] Buddhism
    [C] Jainism
    [D] Islam
    Correct Answer: A [Hinduism]
    Explanation: Hinduism, particularly Shaivism, was prevalent among the rulers of Odisha during the Ganga period, as evidenced by the construction of Hindu temples.
  369. What were the main commodities exported during the Ganga period?
    [A] Spices
    [B] Textiles
    [C] Gemstones
    [D] All of the above
    Correct Answer: D [All of the above]
    Explanation: During the Ganga period, Odisha was known for exporting a variety of goods including textiles, gemstones, spices, and other commodities.
  370. Which period saw the zenith of the Kalinga school of architecture?
    [A] Gajapati
    [B] Karrani
    [C] Bhoi
    [D] Ganga
    Correct Answer: D [Ganga]
    Explanation: The Kalinga school of architecture reached its peak during the Ganga period, showcasing intricate temple designs and artistic excellence.
  371. Who built the Parashurameshvara Shiva temple in Bhubaneswar?
    [A] Gajapati rulers
    [B] Sailodvabhavas
    [C] Bhoi dynasty
    [D] Karrani rulers
    Correct Answer: B [Sailodvabhavas]Explanation: The Sailodvabhavas constructed the Parashurameshvara Shiva temple in Bhubaneswar, indicating their devotion to Shaivism.
  372. Which religious tradition did the Sailodvabhavas contribute to significantly?
    [A] Buddhism
    [B] Jainism
    [C] Shaivism
    [D] Vaishnavism
    Correct Answer: C [Shaivism]Explanation: Sailodvabhavas made notable contributions to Shaivism, as evidenced by their construction of Shiva temples.
  373. During the Ganga period, what was the primary architectural style employed in temple construction?
    [A] Dravidian
    [B] Nagara
    [C] Vesara
    [D] Hoysala
    Correct Answer: B [Nagara]
    Explanation: The Nagara style of architecture was predominantly used during the Ganga period for temple construction in Odisha.
  374. Which dynasty saw the flourishing of trade relations with Southeast Asian countries?
    [A] Bhoi
    [B] Gajapati
    [C] Karrani
    [D] GangaCorrect
    Answer: B [Gajapati]
    Explanation: The Gajapati dynasty facilitated significant trade with Southeast Asian countries, contributing to the prosperity of Odisha.
  375. In addition to textiles and gemstones, what other commodity was exported during the Ganga period?
    [A] Timber
    [B] Ivory
    [C] Rice
    [D] TeaCorrect
    Answer: B [Ivory]
    Explanation: Along with textiles and gemstones, ivory was also exported during the Ganga period, contributing to Odisha’s trade relations with Southeast Asia.
  376. Who led the revolt against the Sunset Law introduced by Cornwallis?
    [A] Surendra Sai
    [B] Chandan Hajuri
    [C] Ramakrushna Samantasinghar
    [D] None of the above
    Correct Answer: C [Ramakrushna Samantasinghar]
    Explanation: Ramakrushna Samantasinghar led the revolt against the Sunset Law, which was introduced by Cornwallis.
  377. In which year was the Odisha State People’s Conference organized in Cuttack?
    [A] 1920
    [B] 1921
    [C] 1929
    [D] 1931
    Correct Answer: D [1931]
    Explanation: The Odisha State People’s Conference was organized in 1931 in Cuttack, with Bhubananda Das serving as the president, to address public grievances.
  378. Who was the president of the Odisha State People’s Conference held in Cuttack in 1931?
    [A] Surendra Sai
    [B] Ramakrushna Samantasinghar
    [C] Bhubananda Das
    [D] Chandan Hajuri
    Correct Answer: C [Bhubananda Das]
    Explanation: Bhubananda Das served as the president of the Odisha State People’s Conference held in Cuttack in 1931.
  379. Which law did Ramakrushna Samantasinghar revolt against?
    [A] Rowlatt Act
    [B] Sunset Law
    [C] Vernacular Press Act
    [D] Pitt’s India Act
    Correct Answer: B [Sunset Law]
    Explanation: Ramakrushna Samantasinghar revolted against the Sunset Law introduced by Cornwallis.
  380. What was the purpose of the Odisha State People’s Conference organized in 1931?
    [A] To discuss educational reforms
    [B] To address public grievances
    [C] To celebrate a historical event
    [D] To promote cultural activities
    Correct Answer: B [To address public grievances]
    Explanation: The Odisha State People’s Conference held in 1931 aimed to address public grievances and concerns.
  381. What recent project did INTACH introduce in Odisha’s Koraput region?
    [A] Revitalizing the Kerang fabric of the Gadaba tribe
    [B] Constructing new homes for tribal communities
    [C] Establishing a museum showcasing tribal artifacts
    [D] Implementing a healthcare initiative for tribal populations
    Correct Answer: A [Revitalizing the Kerang fabric of the Gadaba tribe]
    Explanation: INTACH recently launched a project aimed at revitalizing the Kerang fabric tradition of the Gadaba tribe in Odisha’s Koraput region.
  382. What is the traditional method used by the Gadaba and Bonda tribes to make clothes from Kerang fiber?
    [A] Spinning
    [B] Weaving
    [C] Knitting
    [D] Crocheting
    Correct Answer: B [Weaving]
    Explanation: The Gadaba and Bonda tribes collect bark from wild Kerang trees, extract the fiber, and weave it using traditional methods to make clothes.
  383. What type of clothing do Gadaba women typically wear made from Kerang fabric?
    [A] Shorts
    [B] Sarees with colorful stripes
    [C] Skirts
    [D] Kurtas
    Correct Answer: B [Sarees with colorful stripes]
    Explanation: Gadaba women usually wear full-length clothes or sarees with colorful stripes made from Kerang fabric.
  384. How long are clothes made from Kerang fabric known to last?
    [A] Up to 10 years
    [B] Up to 50 years
    [C] Up to 100 years
    [D] Up to 200 years
    Correct Answer: C [Up to 100 years]
    Explanation: Clothes made from Kerang fabric are known to last for several generations, sometimes up to 100 years.
  385. What was the main objective of INTACH’s project in the Lamptaput block of Odisha?
    [A] Promoting tourism in tribal villages
    [B] Providing healthcare services to tribal communities
    [C] Reviving the tradition of making Kerang fabric
    [D] Offering education and vocational training to tribal youth
    Correct Answer: C [Reviving the tradition of making Kerang fabric]
    Explanation: The main objective of INTACH’s project in the Lamptaput block of Odisha was to revive the dying tradition of making Kerang fabric among the Gadaba tribe by providing assistance and workshops.
  386. In which Indian state has the Swakalpa programme been initiated?
    [A] Odisha
    [B] Karnataka
    [C] Maharashtra
    [D] Tamil Nadu
    Correct Answer: A [Odisha]
    Explanation: The Swakalpa programme has been initiated in the state of Odisha by the Odisha Skill Development Authority (OSDA) in collaboration with Meesho and other partners.
  387. What is the vision of the Swakalpa programme?
    [A] To provide free healthcare to rural communities
    [B] To transform Odisha into the Entrepreneurial Capital of the Country
    [C] To promote eco-tourism in the Himalayan region
    [D] To enhance agricultural productivity in rural areas
    Correct Answer: B [To transform Odisha into the Entrepreneurial Capital of the Country]
    Explanation: The vision of the Swakalpa programme is to transform Odisha into the Entrepreneurial Capital of the Country.
  388. How many individuals are expected to benefit from the Swakalpa programme?
    [A] 5,000
    [B] 10,000
    [C] 15,000
    [D] 20,000
    Correct Answer: B [10,000]
    Explanation: The Swakalpa programme aims to benefit 10,000 individuals in Odisha.
  389. What percentage of women representation is targeted in the Swakalpa programme?
    [A] 10%
    [B] 15%
    [C] 20%
    [D] 25%
    Correct Answer: D [25%]
    Explanation: The Swakalpa programme aims for 25% women representation among its participants.
  390. How many hours of entrepreneurship training are provided under the Swakalpa programme?
    [A] 50 hours
    [B] 75 hours
    [C] 100 hours
    [D] 125 hours
    Correct Answer: C [100 hours]
    Explanation: Participants receive 100 hours of entrepreneurship training as part of the Swakalpa programme.
  391. How long is the mentorship period provided to participants after the entrepreneurship training?
    [A] 3 months
    [B] 6 months
    [C] 9 months
    [D] 12 months
    Correct Answer: B [6 months]
    Explanation: Participants receive six months of mentorship and handholding support after completing the entrepreneurship training.
  392. Which prominent e-commerce platform has partnered with OSDA to implement the Swakalpa programme?
    [A] Amazon
    [B] Flipkart
    [C] Meesho
    [D] Snapdeal
    Correct Answer: C [Meesho]
    Explanation: Meesho, in partnership with OSDA, is managing the implementation of the Swakalpa programme in Odisha.
  393. What is Muruja?
    A. A form of pottery
    B. A type of embroidery
    C. A contemporary art form similar to Rangoli
    D. A style of mural painting
    Answer: C. A contemporary art form similar to Rangoli
  394. What materials are used to create Muruja?
    A. Clay and charcoal
    B. Powders of different hues such as white, green, black, yellow, and red
    C. Oil paints and canvas
    D. Fabric and thread
    Answer: B. Powders of different hues such as white, green, black, yellow, and red
  395. Which town in Odisha is famous for its applique work?
    A. Cuttack
    B. Bhubaneswar
    C. Pipili
    D. Konark
    Answer: C. Pipili
  396. What is the origin of Pipili’s applique work?
    A. Mughal influence
    B. British colonization
    C. Lord Jagannath culture during the 12th century
    D. Portuguese traders
    Answer: C. Lord Jagannath culture during the 12th century
  397. What are some common forms created using applique craft?
    A. Pottery and ceramics
    B. Sculptures and statues
    C. Birds, animals, flowers, and leaves
    D. Textiles and garments
    Answer: C. Birds, animals, flowers, and leaves
  398. For what occasions are large applique canopies used in Odisha?
    A. Birthdays
    B. Funerals
    C. Marriage celebrations
    D. Religious processions
    Answer: C. Marriage celebrations
  399. What is the significance of tiny mirrors in Odisha’s applique craft?
    A. They reflect sunlight
    B. They represent spiritual enlightenment
    C. They enhance the aesthetic appeal
    D. They ward off evil spirits
    Answer: C. They enhance the aesthetic appeal
  400. Which traditional colors are commonly used in Odisha’s applique craft?
    A. Blue, green, purple, and orange
    B. Red, black, yellow, and white
    C. Pink, brown, gray, and teal
    D. Gold, silver, bronze, and copper
    Answer: B. Red, black, yellow, and white

Foundation Course For Odisha Police SI & Constable Exam 2024 | Online Live Classes by Adda 247

OSSSC COMBINED Batch For Category of Teachers Under ST & SC Department | Online Live Classes by Adda 247